Download as docx, pdf, or txt
Download as docx, pdf, or txt
You are on page 1of 92

LỜI GIỚI THIỆU

Tổng hợp Ngân hàng câu hỏi ôn tập môn Thực hành tiếng Anh 3.1 gồm:

Phần 1 – Từ vựng: 170 câu hỏi thi

Phần 2 – Ngữ pháp: 150 câu hỏi thi

Phần 3 – Đọc hiểu: 240 câu hỏi thi

Phần 4 – Nghe hiểu: 195 câu hỏi thi

Phần 5 – Viết: 30 đề thi tự luận

Phần 6 – Nói: 31 đề thi tự luận

Tổng số: 755 câu hỏi thi trắc nghiệm và 61 đề thi tự luận

1
MỤC LỤC

MÔ TẢ ĐỀ THI……………………………………………………………………………………….3

NGÂN HÀNG CÂU HỎI ÔN THI TRẮC NGHIỆM……………………………………………....4

PART 1: VOCABULARY………………………………………………………………….....4

PART 2: GRAMMAR………………………………………………………………………..11

PART 3: READING………………………………………………………………………….17

PART 4: LISTENING………………………………………………………………………..42

NGÂN HÀNG CÂU HỎI ÔN THI TỰ LUẬN……………………………………………………..53

PART 1: WRITING………………………………………………………………………….53

PART 2: SPEAKING………………………………………………………………………...56

ANSWER KEY……………………………………………………………………………………….87

2
MÔ TẢ ĐỀ THI

Thời gian thi: 130 phút


Trắc nghiêm máy: 90’ (Nghe: 40’; Đọc: 50’)
Nói: 10’
Viết: 30’

Số câu thi:
Trắc nghiệm máy: 120 câu
Nói: 3 phần
Viết: 1 đoạn văn

Định dạng bài thi:


Trắc nghiệm máy
- Đọc: 80 câu
Phần 1: 30 câu từ vựng
Phần 2: 25 câu ngữ pháp
Phần 3: 10 câu chọn A, B, C, D điền chỗ trống cho 1 văn bản
Phần 4: 15 câu đọc hiểu chọn A, B, C, D

- Nghe: 40 câu (nghe 2 lần)


Phần 1: 20 câu chọn A, B, C, D (nghe 4 đoạn)
Phần 2: 20 câu điền thông tin (nghe 4 đoạn)

Tự luận
- Viết: 30 phút: Viết một đoạn văn khoảng 150 từ
- Nói (trực tiếp): gồm 3 phần: phỏng vấn, trình bày chủ đề, thảo luận

3
NGÂN HÀNG CÂU HỎI ÔN THI TRẮC NGHIỆM

PART 1 – VOCABULARY

Read the following sentences and choose the best answer for the blank.

1. I am looking forward to our university ________ to Ha Long Bay.


A. trip B. voyage C. travel D. excursion
2. The hotel where we are ________ is quite luxurious.
A. living B. staying C. existing D. remaining
3. I asked him to ________ outside the Mega Mall.
A. let me out B. let me get out of C. drop me off D. drop me in
4. Mr. Smith was fetching the luggage while Mrs. Smith was ________ at the resort.
A. signing in B. giving in C. pulling in D. checking in
5. Why don’t we ________ the scenic route along the coast?
A. go to B. take C. catch D. come
6. I’m sorry I’m late! I ________ my way and had to ask for directions.
A. lost B. missed C. leave D. misplace
7. When you ________ your destination, your tour guide will meet you at the airport.
A. reach B. get C. arrive D. achieve
8. I regret ________ at the hotel view before I booked it.
A. to look B. not to look C. looking D. not looking
9. I have already searched for a ________ flight from London to Hanoi on November 23rd but
unfortunately, there isn’t any one that day.
A. direction B. directive C. direct D. indirect
10. Hanoi has changed so much in the last few years that it’s almost ________.
A. recognized B. recognizable C. unrecognizable D. recognition
11. We used to go skating at the ice ________ every weekend.
A. ring B. rink C. court D. field
12. We got through the ________, but we lost to Tottenham.
A. final B. finale C. end D. ending
13. I don’t know how you can stand getting up so early to go to the pool. Stand here is similar to
________.
A. put off B. join in C. put up with D. go off
14. Why do you ________ having riding lesson if you can’t afford them?
A. carry on B. go in for C. get round to D. keen on
15. Why don’t you ________ a go? It is not difficult.
A. make B. get C. take D. have
16. Our new coach is popular ________ the whole team.
A. with B. to C. for D. from
17. Before we play, ________ certain all the cards are there.
A. do B. give C. make D. take
18. It can be quite busy here during the tourist ________.
A. season B. period C. phase D. stage
19. When I first started playing golf, I have no real ________ of the rules.
A. acknowledge B. know C. knowledge D. knowledgeable
20. Would you describe yourself as a ________ person?
A. competition B. competitive C. compete D. competitor
21. The ________ from London to Berlin is about 919 kilometres.
A. measure B. length C. distance D. gap
22. Individual scientists often received very little ________ for their hard work.
A. reward B. gift C. award D. prize
23. A truck had ________ on the highway so we were stuck in a traffic jam.

4
A. come down B. broken down C. given down D. turned down
24. Wear a mask because these chemicals ________ fumes that can be harmful.
A. turn off B. come off C. give off D. cut off
25. Communication changed completely ________ the introduction of the telephone.
A. by B. with C. from D. birth
26. The police are investigating the ________ of the explosion.
A. reason B. motive C. base D. cause
27. The mistake by scientists resulted ________ a massive explosion.
A. in B. on C. of D. from
28. Dr Pham finally ________ discover the secret formula.
A. planned to B. managed to C. looked forward to D. looked up to
29. The ________ of a unique wireless Internet connection means there’s a world of discovery just
waiting for you.
A. invent B. inventor C. invention D. inventive
30. I need to consult a ________ about my anxiety.
A. psychic B. psychology C. psycho D. psychologist
31. We interrupt this movie to bring you an urgent ________.
A. newsflash B. bulletin C. feature D. article
32. On TV, a/an ________ tells you what the next programme is going to be.
A. commentator B. journalist C. announcer D. columnist
33. There was a woman in the mall ________ vouchers to that new spa.
A. bringing out B. handing out C. making out D. turning out
34. I can ________ through your lies.
A. look B. smell C. sense D. see
35. The newspaper is now ________ the control of Robert Langdon.
A. under B. after C. out D. in
36. In my ________, immigrations shouldn’t be banned in US.
A. thought B. sight C. view D. mind
37. I think you’re confused tabloids ________ broadsheets.
A. to B. with C. from D. by
38. We all surprised ________ the news that her grandfather has a cancer.
A. by B. to C. from D. of
39. Why do the paparazzi think they can turn up at a movie star’s house completely ________?
A. announced B. unannounced C. announcement D. announcer
40. There is a/an ________ rule on tabloid newspapers that the truth always takes the second place to
a good story.
A. writing B. written C. unwritten D. wrote
41. The cat that lives by the river has ________ all of my efforts to make friend with her.
A. hailed B. resisted C. swept away D. transformed
42. If you buy a large drink at the movie theater, it is ________ that you won’t finish the whole
thing.
A. a handful of B. a good bet
C. more often D. seems to be the case
43. Most people agree that the older ________ of the movie is better than the newer one.
A. masterpiece B. critic C. screen D. version
44. The actors in the movie are famous, but I’ve never heard of the ________.
A. adaptation B. director C. genre D. medium
45. Movies have the ability to ________ the way our society thinks about certain issues.
A. impact B. measure C. inform D. continue
46. It was a fun book to read, but I don’t think it has much ________ value.
A. impossible B. literary C. mediocre D. perhaps
47. You didn’t win the competition, but I ________ your effort.
A. applaud B. adore C. fulfill D. measure up
48. Researchers believe this new medicine could ________ save thousands of lives each year.
5
A. individually B. traditionally C. particularly D. potentially
49. The image of Italians shown in the movie ________ reality.
A. finds out B. is far from C. is sure to be D. woe be it
50. The news broadcast reported on the ________ danger to coastal cities due to the approaching
hurricane.
A. depending B. thrilling C. impending D. migrating
51. Personally, I’m not ________ to believe in omens or superstitions.
A. blessed B. found C. inclined D. inspired
52. The sign by the road warned drivers to ________ of falling rocks.
A. accompany B. beware C. clip D. originate
53. He wrote a book comparing ________ about colours between Eastern and Western cultures.
A. indications B. infections C. superstitions D. scavengers
54. A common ________ says that acne is caused by a poor diet. This, however, is far from true.
A. over time B. ill fortune C. crown of England D. old wives’ tale
55. She introduced Sally to me, and then she ________ us on our blind date.
A. accompanied B. diminished C. reproduced D. sealed
56. Doctors say the new vaccine could ________ malaria.
A. be far from B. put up with C. turn up D. wipe out
57. We can’t measure the molecule’s size exactly, but we can ________ it.
A. arise B. beware C. estimate D. thaw
58. Did you know there’s a(n) ________ on top of Mount Kilimanjaro?
A. activity B. dilemma C. glacier D. increase
59. It is sometimes difficult for animals to ________ if their homes are destroyed.
A. browse B. erode C. maintain D. relocate
60. Each American eats a(n) ________ of 50 pounds of chocolate per year.
A. average B. gradual C. initial D. proper
61. Both of my grandparents were born at the ________.
A. tried and true B. old wives’ tale C. turn of the century D. gradual
62. It is hard to ________ on my homework when people are talking loudly.
A. recommend B. focus C. workout D. benefit
63. If I get a ________ score on my final exam, my mom said she would buy me a new watch.
A. perfect B. proper C. balanced D. short
64. If you want to ________ from regular exercise, you should also eat healthy foods.
A. prevent B. progress C. benefit D. achieve
65. Ben’s broken leg is ________ him from going on the bicycle trip.
A. rushing B. preventing C. eroding D. reproducing
66. She is a(n) ________ at teaching yoga, and I have learnt a lot from her classes.
A. expert B. development C. routine D. exercise
67. It is very important to eat a ________ diet that contains fruits, vegetables, and whole grains.
A. perfect B. common C. balanced D. rare
68. The plan we made were too difficult, so we could not ________ it.
A. bear in mind B. wipe out C. turn up D. stick to
69. The actor’s last movie ________ his best work.
A. was far from B. was fun to C. was traced back D. was wiped out
70. The committee suggested a ________ plan to deal with the problem, but the plan won’t solve
things completely.
A. physical B. little C. short-term D. higher
71. The process was too more complicated than I had ________ thought.
A. initially B. literally C. permanently D. physically
72. In the next chapter, we will learn about the ________ of the moon around the Earth.
A. dilemma B. goal C. mass D. motion
73. The survey results showed that ________ who drive have relatively few accidents.
A. abilities B. babies C. rhythms D. seniors
74. The company installed a firewall to ________ its computer security.
6
A. beef up B. open up C. put up with D. turn up
75. The city crime ________ has dropped five percent over the last year.
A. goal B. pressure C. rate D. therapy
76. The book message was very powerful. It has a ________ power for readers.
A. transformation B. transformed C. transformative D. transformer
77. He used ________ parts from other old cars to rebuild his 1957 Chevy.
A. scavenger B. scavenged C. scavenging D. scavenge
78. The ocean constantly reshapes coastlines by ________ the beaches, cliffs, and land.
A. erosion B. erode C. eroded D. eroding
79. Please ________ during the lesson, or you will not remember anything for the test.
A. concentrate B. concentration C. concentrating D. concentrator
80. Being more ________ active is one of the best things you can do for yourself.
A. physically B. physical C. physique D. physics
81. For a start, I was very _______ and I think that animals can sense it when you’re nervous and
worried.
A. excited B. happy C. sad D. anxious
82. Actually, I was extremely ________ when the lesson finished.
A. sad B. interested C. impressed D. relieved
83. I came second so I was really ________.
A. full B. empty C. awkward D. thrilled
84. I was absolutely ________ to see how different people behaved when they got in front of the
microphone.
A. worried B. scared C. sad D. fascinated
85. It took me ages to do and I got really annoyed and ________ at one point because I couldn’t
make it straight.
A. sad B. happy C. funny D. frustrated
86. I felt really ________ when I’d finished because it was the first tent I’d ever put up on my own.
A. thrilled B. sad C. impressed D. satisfied
87. Most people were there for the first time and I was really ________ by how quickly they learnt
the steps.
A. impression B. nervous C. anxious D. impressed
88. I was very ________ because I kept treading on her toes.
A. excited B. interested C. thrilled D. embarrassed
89. I suggested calling the repair company. I felt very _____ about it but I thought we’d never get
home.
A. tired B. exhausted C. funny D. awkward
90. I didn’t get to bed till two in the morning and I was completely ________.
A. good B. excited C. sleepy D. exhausted
91. Lena’s a good worker. She ________ around the office.
A. pushes her weight B. pulls her height C. pulls her weight D. pushes her leg
92. Tessa ________ leaving her desk tidy at the end of the day.
A. pays attentions to B. is interested C. is particular D. is excited
93. My friend is really ________. She always has time for me when I have any problems. She seems
to understand and wants to help me.
A. cautious B. naive C. sympathetic D. witty
94. I was with my husband when we ran into his ex-wife. It was very ________.
A. boring B. exciting C. awkward D. fascinating
95. Beth thought she’d lost her passport so she was incredibly________ to find it.
A. shocked B. worried C. relieved D. nervous
96. The boss was ________ with my work on the report and she’s giving me tomorrow off.
A. sad B. anxious C. satisfied D. worried
97. You can tell Marta your secret and you know she’ll never tell anyone else. She is such a _______
person.
A. flexible B. practical C. trustworthy D. confident
7
98. A ________ deposit is required when booking the holiday and you can’t get it back if you cancel
your booking.
A. credit B. payable C. non-refundable D. limited
99. We are going to try you on this new treatment and we’ll _______ your progress carefully.
A. look B. research C. monitor D. approve
100. Corruption isn’t only deeply ________, it is also ________.
A. disturbing / inoffensive B. justifiable / unethical
C. disturbing / unethical D. economic / political
101. Are full body scanners at airports a/an ________ of privacy?
A. right B. invasion C. reason D. way
102. Hunting elephants is ________, but hunting deer is allowed, with a permit.
A. legal B. illegal C. good D. permitted
103. Buying an essay from the Internet and saying you wrote it is totally________.
A. impractical B. unethical C. insensible D. imperfect
104. It was absolutely________ that the workers were told of their job losses by text message.
A. unethical B. outrageous C. illegal D. ignored
105. I found the programme about child beauty competitions slightly ________. It made me feel
anxious.
A. disgusting B. disturbing C. shocking D. interesting
106. How ________ is it to buy medicine over the Internet from someone you don’t know?
A. economical B. sensible C. ethical D. legal
107. Eric simply ________ all the complaints from his lodgers. He paid no attention to them.
A. talked B. ignored C. blamed D. spoke
108. Increasingly, Naomi ________ her work because of all her other responsibility.
A. neglected B. complimented C. paid attention D. set up
109. This problem has nothing to do with other countries. It’s a purely ________ issue.
A. domestic B. international C. overseas D. foreign
110. Sam ________ whenever his wife gave him a good advice. He listened to her carefully.
A. paid attention B. ignored C. neglected D. set up
111. Sue ________ of all the administrative details.
A. takes care B. takes over C. takes up D. takes on
112. The situation can’t be decided by one or two countries acting alone. It’s a ________ issue.
A. global B. national C. nationwide D. domestic
113. You can be so________. You know you were wrong, you should say sorry.
A. stubborn B. nice C. mean D. generous
114. The company has________ more than 50 new workers.
A. taken on B. taken up C. taken down D. taken over
115. We have ________ a number of new business centers in major cities.
A. set up B. look down C. get over D. lay down
116. We wanted to find a house ________ to the lake.
A. in close proximity B. near C. next to D. behind
117. The houses that were ________ of the lake were too expensive.
A. on the edge B. the left C. the right D. behind
118. That was a very ________ thing to do. You never think of anyone else.
A. selfish B. wise C. good D. wonderful
119. Domestic ________ is a growing problem in times of unemployment.
A. divorce B. violence C. drunkenness D. obesity
120. The show is only on a________, so let’s see it as soon as possible.
A. limited run B. limited enrolment C. first come first served D. limited edition
121. Would it be possible for people in your town to give up their ________ cars and use only public
transport?
A. individual B. state C. private D. government
122. - Has malaria ___ in recent years? - Yes, there’s been a huge decrease because of the use of nets.
A. decreased B. rose C. increased D. grown
8
123. How ________ is your spelling in your language and in English?
A. accuracy B. falseness C. trueness D. correction
124. How often are you ________ from English lessons or from work due to bad health?
A. present B. absent C. existing D. unconscious
125. The country has serious __ problems, which mean that taxes will probably have to double next
year.
A. economic B. economical C. politic D. political
126. Corruption isn’t only deeply ________, it’s also ________.
A. disturbing/inoffensive B. justifiable/ unethical
C. disturbing/unethical D. justifiable/inoffensive
127. We need to________ all their comings and goings over the next twenty-four hours.
A. log B. keep track C. permit D. admit
128. I felt some________ when I couldn’t make myself understood in Spanish.
A. frustrating B. frustration C. frustrated D. frustrate
129. John is a ________ person. He always takes a lot of time to make up his mind and he’s careful to
avoid problems or danger.
A. moody B. cautious C. eccentric D. mean
130. Do you think people should get a ________ to pay a sum of money for driving too fast or should
they lose their licence?
A. punishment B. reminder C. fine D. criticism
131. Is it better to have a boring but ________ job for life or a number of short-term jobs that you
love?
A. temporary B. compulsory C. permanent D. optional
132. Joe was so ________ when he thought that his first job would be easy and people would be nice
and then he soon learnt differently.
A. naive B. genuine C. flexible D. careless
133. My mum was________ thrilled to hear the news.
A. very B. fairly C. absolutely D. likely
134. It is ________ that there will be job losses. It’s going to happen sooner or later, that’s for sure.
A. evitable B. inevitable C. legal D. illegal
135. A: They really shouldn’t have closed the school.
B: I agree ________, but there were good reasons for it.
A. for some extent B. to some extent C. to some point D. for some point
136. The president’s ________ for the early release of the hostages has not been successful.
A. sponsor B. appeal C. absence D. presence
137. Some say that a guard dog is the best________ to crime.
A. deterrent B. treatment C. surveillance D. prevention
138. Australia is the least densely ________ with just 2.6 people per square kilometer.
A. polluted B. populous C. populated D. popularized
139. The most remote and unspoilt place in the UK is Foula, an island just off the _____ of northern
Scotland.
A. coast B. sea C. beach D. shore
140. We reached the top of the hill but the climb was completely ________. We all felt so tired.
A. thrilling B. exhausting C. surprising D. amazing
141. She’s been feeling a lot of________ about the new job – that’s why she can’t sleep.
A. generosity B. anxiety C. satisfaction D. disappointment
142. ________, the minimum voting age should be sixteen.
A. The way I see it B. I’m against C. It seems to me as D. It seems
143. The most ________ way forward seems to be to organize a meeting to discuss the issue calmly.
A. sensible B. inevitable C. outrageous D. sensitive
144. Her ______ was obvious as she read her exam results. They were much lower than she expected.
A. expectation B. satisfaction C. disappointment D. excitement
145. They are shipping in food, but thousands will still die of hunger in the ________.
A. obesity B. debt C. famine D. flood
9
146. He never pays for anything, he’s so ________.
A. tight-fisted B. generous C. down to earth D. kind
147. You got the highest mark in the exam. I’m really ________.
A. exhausted B. anxious C. impressed D. interested
148. The lack of rain this summer may cause an autumn ________.
A. poverty B. drought C. disaster D. earthquake
149. Number plate ________ helps the police to keep track of stolen cars.
A. recognition B. registration C. discovery D. enrolment
150. Never ________ something ________ until tomorrow if you can do it today.
A. put / on B. put / off C. put / back D. put / into
151. The way it works is that you ________ the dice to see who starts.
A. shake B. bounce C. roll D. throw
152. If the computer freezes, just ________ it and wait a few minutes.
A. unplug B. press C. jam D. hold
153. Sorry, I must go. I have to on with an urgent report.
A. get B. give C. take D. put
154. The poor dog was hungry and suffering from ________. Nobody took care of it.
A. ignorance B. consideration C. neglect D. carelessness
155. I’m sorry you didn’t like what I said, but ________ it!
A. get over B. step down from C. put off D. put down
156. He shouldn’t always ________ in to his boss’s demands. It is not a good idea for him to change
his opinion at all times.
A. take B. give C. get D. go
157. ________ is still illegal in many countries. You are not allowed to end your marriage.
A. Divorce B. Domestic violence C. Fighting D. Breaking laws
158. We’re getting worried because they haven’t ________ yet and it’s getting dark.
A. set in B. turned up C. settled down D. turned down
159. The sight of teenage drunkenness on the streets is really ________.
A. acceptable B. cultural C. offensive D. unethical
160. The level of poverty in many parts of the world is _______.
A. measurable B. unjustifiable C. justifiable D. immeasurable
161. It’s ________ that people in rich countries ignore the existence of famine and drought in the
world.
A. outrageous B. ethical C. possible D. sympathetical
162. Law-abiding should feel safer living in a surveillance society.
A. customers B. citizens C. clients D. participants
163. He took a photo of the shoplifter with his phone camera and it over to the police.
A. helped B. handled C. handed D. hung
164. How high is the ________ of the mountain?
A. summit B. slope C. edge D. center
165. Whenever I ask him to do something he just ________ me.
A. neglects B. ignores C. doesn’t pay attention D. doesn’t listen
166. Stop complaining and just ________ with the job!
A. don’t put off B. give in C. get on D. get off
167. Without enough exercise, an increase in ________ is inevitable.
A. malnutrition B. weightlessness C. obesity D. nutrition
168. The more you ________ the more stubborn she gets.
A. compliment B. criticize C. congratulate D. adore
169. The decision on the election date is a/an _____ question and will depend on the government’s
popularity.
A. global B. industrial C. political D. regional
170. Noniko is a great addition to the team. She is ________. She adapts quickly to the new situations
and doesn’t mind change.
A. witty B. flexible C. confident D. independent
10
PART 2 – GRAMMAR

Read the following sentences and choose the best answer for the blank.

1. Look out! You ________ hit the car in front!


A. are going to B. will C. are D. did
2. I ________ do the washing-up tonight if you like.
A. am going to B. will C. am D. will be
3. Do you think that in the future people ________ to be hundreds of years old?
A. are going to live B. are living C. live D. will live
4. Elizabeth________ to bed at round eleven o’clock.
A. usually goes B. is usually going C. go D. going
5. Dan ________ on the other phone right now.
A. talking B. is talking C. talked D. talks
6. We ________ any meat at the moment as were are both on a diet.
A. don’t eat B. aren’t eating C. not eat D. didn’t eat
7. My mum ________ me every weekend without fail.
A. is calling B. called C. calls D. is going to call
8. No, the train ________ at Cirencester on Saturdays.
A. does stop B. is stopping C. stopped D. will stop
9. My mum ________ part in ice-skating competitions almost every weekend.
A. took B. is taking C. takes D. will take
10. My dad ________ late on Saturday mornings.
A. is often get up B. often gets up C. is getting often D. will often get up.
11. Actually Darren ________ quite hard sometimes.
A. does works B. do works C. works D. worked
12. It is the first time I ________ a party at home.
A. ever having B. have ever had C. ever have D. had ever
13. Have you finished ________?
A. yet B. already C. still D. just
14. It is eleven o’clock and Todd ________ hasn’t come home.
A. still B. yet C. just D. recently
15. That is the first time ________ an answer right today!
A. I get B. I am getting C. I have got D. I have been getting
16. Jessica has ________ left, I am afraid.
A. already B. yet C. still D. so far
17. Dan ________ in the living room while we are redecorating his bedroom.
A. sleeps B. is sleeping C. has slept D. does sleep
18. Actually, I ________ a cup of tea first thing every morning but then I switch to coffee.
A. did drink B. am drinking C. have drunk D. have been drinking
19. I ________ to all the local newspapers and TV stations to complain.
A. already write B. already writing
C. have already written D. have already been writing
20. Eric, ________ hockey competitively or just for fun?
A. do you usually play B. are you usually playing
C. have you usually played D. have you usually been playing
21. My brother and I ________ swimming almost every day last summer.
A. went B. had been going C. were going D. had gone
22. We ________ when someone knocked at the door.
A. talked B. had talked C. were talking D. were talked
23. When the robbery happened, the security guard ________!
A. slept B. was sleeping C. had slept D. was slept
24. Jack ________ chess before I showed him what to do.
A. hadn’t been playing B. didn’t play C. wasn’t playing D. hadn’t played
11
25. I wasn’t sure how Belinda would react because I ________ her long.
A. didn’t know B. wasn’t knowing C. hadn’t been knowing D. hadn’t known
26. Ian ________ at the factory long when he was made a manager.
A. hadn’t been working B. wasn’t working C. didn’t work D. wasn’t worked
27. I wanted to say goodbye to Jerry, but he________.
A. was already left B. already left
C. had already been leaving D. had already left
28. When we got to the airport, I realised I ________ my passport at home!
A. was left B. had left C. left D. had been leaving
29. The MP asked ________ the prime minister was aware of the growing social problem.
A. if B. that C. him D. what
30. Here’s ________ DVD you asked to borrow.
A. a B. an C. the D. no word
31. We’re out of ________ coffee, so could you get some from the supermarket?
A. a B. an C. the D. ---
32. The prize is ________ unique opportunity to travel the world.
A. a B. an C. the D. ---
33. If I’d known you were coming, I ________ a cake.
A. would bake B. will bake C. would have baked D. baked
34. It’s ________ honour to be here this evening to speak to you.
A. a B. an C. the D. ---
35. If you see Nina on Friday, ________ her to give me a ring.
A. tell B. you will tell C. tell to D. you would tell
36. Reports are coming in of a major oil spill in ________ Mediterranean.
A. a B. an C. the D. ---
37. I went to see the doctor because I’m finding it difficult to sleep at ________ night.
A. a B. an C. the D. ---
38. Do you think that they’ll ever send a manned mission to ________ Venus?
A. a B. an C. the D. ---
39. Local residents object ________ the new power station in their area.
A. to have B. to having C. of having D. having
40. There is ________ coffee left, if anyone wants some.
A. few B. a few C. a little D. little
41. Jane had to go home from school because she had ________ headache.
A. some B. the C. a D. any
42. The robber was sent to ________ prison for a total of three years.
A. a B. the C. few D. ---
43. My father works at the university, doing research ________ weather control.
A. to B. of C. from D. on
44. The underground doesn’t run this late, so we’ll have to take ________ taxi.
A. a B. the C. other D. these
45. My favourite subject at school is________ chemistry.
A. the B. a C. --- D. another
46. Jazz music appeared in America in ________ 1920s.
A. no word B. the C. these D. those
47. They should arrive ________ the airport in about an hour.
A. at B. in C. on D. off
48. We should arrive ________ Paris at six in the morning.
A. at B. on C. out D. in
49. The universe ________ continue to expand for billions of years.
A. will B. is going to C. shall D. did
50. ________I carry some of those bags for you, or can you manage?
A. Did B. Am C. Will D. Shall
51. This time next week, we ________ the chemistry exam.
12
A. have finished B. have been finishing
C. will have finished D. will have been finishing
52. When I grow up, ________ an inventor.
A. I’m being B. I’m going to be C. I will have been D. I will be being
53. Shirley will ________ her research for the next few weeks.
A. have done B. be doing C. have been doing D. have been done
54. Next year, Sam will ________ patients at this hospital for twenty-five years.
A. have been treating B. treat C. be treating D. be going to treat
55. What is it? We won’t know until ________ at it under a microscope.
A. we are going to look B. we’ll have looked C. we’ll be looking D. we’ve looked
56. You won’t get any radio reception while ________ through the tunnel.
A. you’ve driven B. you’ll drive C. you’re driving D. you’ll be driving
57. - I’m going to set up the equipment in a minute. - ________ give you a hand?
A. Shall I B. Will I C. Would I D. Do I
58. How much ________ the bus to the city center?
A. are B. is C. cost D. does
59. You can watch the film ________ you promise to go straight to bed when it finishes.
A. as if B. unless C. in case D. as long as
60. What did you use to do on Sundays? We would usually ________ to church in the morning.
A. go B. went C. were going D. used to go
61. I ________ reading a wonderful book about space travel.
A. just finish B. just do finish
C. have just finished D. have just been finishing
62. It was the first time we ________ to a Mexican restaurant.
A. go B. went C. have been D. had been
63. - Can you come out to play? - No, we ________ dinner at the moment.
A. have B. are having C. have had D. have been having
64. How much ________ to get into the arena?
A. it costs B. costs it C. does it cost D. does it costs
65. It’s only the second time I ________ a job interview.
A. have B. am having C. have had D. have been having
66. What did you do while ________ for the others to come?
A. were you waiting B. you were waiting
C. had you been waiting D. you had been waiting
67. Did you think Sam ________ just like his father?
A. looks B. is looking C. has been looking D. had been looking
68. I promise I ________ pay you soon.
A. am going to B. will C. did D. am
69. She ________for 5 hours. She seems to be tired now.
A. has been working B. has working C. worked D. had worked
70. By the end of the month we ________ 5 assignments.
A. will finish B. finish C. will have finished D. have finished
71. Could you tell me what time ________?
A. the film starts B. does the film starts C. the film does starts D. does the film start
72. Have you ever ________ a plane before?
A. flied B. flow C. flown D. fly
73. Can I ask if you’d ________ a plane before?
A. ever flown B. ever flied C. never flown D. never fly
74. Can I ask you what ________?
A. does the weather like? B. the weather look like?
C. the weather is like? D. is the weather look like?
75. Do you know who ________?
A. he is work for B. is he work for C. is he working for D. he is working for
76. I’d be interested to know how much ________.
13
A. you have to pay for it B. you had to pay for it
C. did you had to pay for it D. you pay for it
77. Do you mind me ________ that wonderful picture?
A. ask you who painted B. asking who painted
C. ask if who painted D. asking who paint
78. I’m so hungry! I ________ for six hours without a break.
A. have worked B. have been worked C. have been working D. had worked
79. She went on a charity walk last week and she ________ £500.
A. has raised B. has been raising C. has rised D. has risen
80. Sorry about the mess. We ________ all week.
A. have decorated B. have decorating
C. have been decorating D. has decorated
81. ________ started his new job yet?
A. Have he B. Has he C. Has he been D. Had he been
82. Somebody ________ my favourite mug! Come on, who was it?
A. has been breaking B. has breaking C. has broken D. has been broken
83. It has been a very productive day. We ________ six cakes and two casseroles.
A. have make B. have made C. have been making D. had been making
84. I ________ any good films recently.
A. haven’t seen B. didn’t see C. haven’t been seeing D. hadn’t seen
85. She ________ reading when she was a girl.
A. loves B. loved C. has loved D. had loved
86. Sarah ________ writing her first novel last week.
A. has started B. has been started C. started D. had started
87. He’s never been very interested in fixing things but lately he ________ quite a few repairs
around the house.
A. has been done B. had been doing C. has done D. had done
88. She ________ the violin in an orchestra for five years before she became a soloist.
A. plays B. has played C. has been playing D. played
89. I’ve learnt more over the past fortnight with our new teacher than I ________ for the previous
two years.
A. do B. did C. done D. have done
90. We ________ that crime drama series for six weeks – the ending was very disappointing.
A. have watched B. watched C. have been watching D. was watching
91. He’s studied more over the past fortnight than he ________ all year.
A. did B. was done C. has done D. had been doing
92. I stopped watching that period drama after two weeks because the acting ________ very
disappointing.
A. has been B. have been C. was D. is
93. She loved cooking when she was younger and ________ her own restaurant.
A. has just opened B. has been opening C. was opened D. was opening
94. They ________ very interested in pop music but recently they’ve been to several gigs.
A. never be B. never were C. have never been D. have been
95. Oh no! My mobile ________!
A. has been stolen B. has stolen
C. has been being stolen D. had stolen
96. The house ________ during the night and valuable jewellery was taken from the safe.
A. was rob B. was robed C. was robbed D. has been robbed
97. They’re too scared to go out. They think they ________.
A. watched B. are watched C. are watching D. are being watched
98. The job must finish by Friday or you ________.
A. will pay B. will be paid C. won’t pay D. won’t be paid
99. Fifty people ________ in a train crash.
A. has injured B. have injured C. have been injured D. has been injured
14
100. Celebrities are followed everywhere they go. They ________ to have a private life. It must be
awful.
A. are allowed B. aren’t allowed C. allows D. allow
101. Be careful what you say. He doesn’t like ________.
A. criticize B. be criticized C. being criticized D. criticizing
102. She’s the sort of person who wants ________ what to do.
A. to be told B. to tell C. to be telling D. being told
103. I ________ in Blackwater. They’ve got everything there!
A. has been shopping B. have been shopping C. shopping D. had been shopping
104. Who has ________ my favourite CD? Come on, where is it?
A. been borrowing B. borrowed C. borrowing D. borrow
105. He swam five miles last week and he has ________ £1000 for charity.
A. raised B. rised C. been raising D. been rising
106. It has been a very productive morning. I ________ the whole report!
A. was writing B. wrote C. have been writing D. have written
107. Sorry about the terrible mess. We’ve ________ all day.
A. cooked B. cooking C. have been cooking D. was cooked
108. Famous people are not allowed to have a private life. They ________ everywhere they go.
A. photograph B. are photographed C. was photograph D. photographs
109. I am phoning ________ a course.
A. to require about B. about enquiring C. enquire about D. for enquiring
110. - What ________? - She’s great. I really like her.
A. is like your new boss B. is your new boss like
C. does you new boss like D. does your new boss look like
111. Teenagers don’t like ________.
A. to be advised B. being advice C. to be advising D. advising
112. The order must deliver by tomorrow or the cheque ________.
A. will cancel B. will be cancelled C. will being cancelled D. will be cancelling
113. We’ve been very lucky ________.
A. so far B. yet C. already D. still
114. I used ________ every day after work.
A. to go running B. going running C. going run D. to going run
115. It took her nearly a year to get ________ alone.
A. used to live B. use to live C. used to living D. use to living
116. When we lived in Spain, we’d ________ late and sleep in the afternoon.
A. going to bed B. go to bed C. went to bed D. gone to bed
117. Be careful. You’re not used to ________ on the right.
A. drive B. to drive C. driving D. to be driven
118. They didn’t use to ________ spicy food but they quite like it now.
A. eating B. eat C. being eaten D. being eating
119. Would there be ________ holding my place.
A. any chance for B. any chance of C. any chance D. any chance with
120. I expect they ________ be late again.
A. are going to B. are C. will D. will be going to
121. They say it ________ to be sunny tomorrow but I think it’ll rain.
A. might B. might be C. is likely D. is going
122. Look! That car ________ crash into the bus!
A. is B. may C. may be D. is going to
123. The library usually ________ at 9 and closes at 5.30.
A. opening B. opens C. be open D. being opened
124. The murderer ________ yet.
A. hasn’t been caught B. hasn’t caught C. wasn’t caught D. didn’t catch
125. You ________ the plane. It leaves at 12.30 and it’s already 12 o’clock.
A. are definitely miss B. are definitely missing
15
C. will definitely miss D. will be definitely miss
126. He’s a very promising footballer. There’s a good chance that he ____ play for England one day.
A. may be B. may well C. will be D. will
127. We didn’t use to ________ Mexican food but we often eat it now.
A. like B. liking C. be liked D. liked
128. He could never ________ the high temperatures in India.
A. used to B. use to C. get used to D. get use to
129. What is the ________ between a clock, a coin and a mountain?
A. similarity B. similar C. similarly D. similarities
130. When they lived in Spain, they got used to ________ late.
A. eat B. eats C. eating D. being eaten
131. In the summer, we would ________ on the beach all day.
A. be playing B. play C. to be played D. playing
132. They ________ the start of the film. It starts at 6.00 and it’s already 5.30.
A. are probably missing B. are probably miss
C. will probably be missed D. will probably miss
133. She’s a brilliant tennis player. There’s a good chance that she _______ at Wimbledon next year.
A. will be played B. will play C. plays D. will playing
134. They say we ________ win but I don’t think we will.
A. might B. might be C. are likely to D. are likely
135. Look! That man ________ walk in front of the bus!
A. is go to B. is going to C. may well D. may be well
136. The museum usually ________ at 10.
A. open B. opening C. opens D. is opened
137. The modern Frisbee ________ by Frisbie Pie Company in 1946.
A. was invented B. were invented C. is invented D. invented
138. I’m taking an umbrella in case it ________.
A. rain B. is raining C. rains D. is rain
139. Paul ________ in God since he was a child.
A. believed B. has believed C. has been believing D. have been believing
140. I couldn’t find a job ________ university.
A. since B. after C. for D. while
141. I’d be interested to know where ________.
A. does they work B. they do work C. they work D. do they work
142. I ________ it until I went to Australia.
A. never see B. never saw C. have never seen D. never seen
143. I ________ find a job since university.
A. haven’t been found B. haven’t been able to C. have been found D. have being able to
144. Three climbers have been reported missing. The mountain rescue team must _____ for them
now.
A. be looking B. look C. be looked D. have been looking
145. The white surface of the Tai Mahal ________ by the pollution and is turning into yellow.
A. has damaged B. has been damaged
C. has been being damaged D. damaged
146. I ________ snow in my life.
A. have never seen B. never see C. never saw D. have no see
147. Could you tell me what time ________?
A. the film starts B. does the film start C. the film does starts D. start the film
148. I ________ quite a few plays last month but most of them were really disappointing.
A. saw B. had seen C. have seen D. see
149. I ________the book, you can have it back.
A. reading B. have been reading C. has been reading D. have read
150. Why are your hands so dirty? - I ________ my bike.
A. repaired B. have repaired C. have been repairing D. has been repairing
16
PART 3 – READING

Part I: Read the passage below and decide which answer A, B, C or D best fits each space.
Passage 1
Archaeologists in England thought they had made an amazing discovery in July 2003, when
tourists on a beach found ancient carvings on a large block of stone. The archaeologists believed that
the discovery of the stone, which had been imported from Norway in the 1980s and used to make a
wall, was (1)________very (2) ________. The carvings of two snakes, a dragon, and other shapes
made the (3) ________in the local (4)________. Experts translated the stone to say, “This stone is for
people who celebrate with fire.”
However, two months later, the archaeologists were (5) ________when the (6) ________of the
carvings was solved by a fifty-year-old local builder, Barry Luxton. The man, who had seen a
photograph in a newspaper, told them that he was actually the one who had made the shapes - in 1995.
Luxton said that over a period of three days in 1995 he had made the carvings for a celebration on a (7)
________beach that was going to be held by a group of druids - people who (8) ________nature.
However, the block didn’t end up being moved to the other beach and was eventually covered by sand.
Recent bad weather blew the sand away, making the carvings (9) ________again. Luxton was
surprised; he really never (10) ________that his work would become so famous.
1. A. basically B. potentially C. preferably D. totally
2. A. main B. dramatic C. significant D. reliable
3. A. headlines B. headings C. titles D. topics
4. A. source B. resource C. post D. media
5. A. shocked B. shocking C. shock D. shocks
6. A. story B. riddle C. implication D. underlying
7. A. neighbor B. neighborhood C. neighboring D.
neighbored
8. A. pray B. celebrate C. bow D. worship
9. A. visible B. obvious C. audible D. sensible
10. A. dreamed B. wanted C. expected D. liked

Passage 2
One of the most terrible environmental disasters of the last fifty years was that which (1)
________ in Minamata Bay in Japan in 1950s and 1960s. Minamata is a rural framing and fishing area
on the Japanese island of Kyushu. It was also the home of a large chemical factory owned by the
Chisso chemical company.
In 1956, a five-year-old girl in the area was taken to the hospital with brain (2) ________ as
well as (3) ________ problems such as being unable to walk, and (4) ________ speech. This girl was
one of the first of thousands of (5) ________ to show similar symptom. Doctors were at first unable to
determine the cause of the problem, which became known as Minamata disease, but finally they
worked it out. Over a long period of time, the Chisso factory had been (6) ________ alkyl mercury (a
chemical) into the bay. This chemical then (7) ________ the fish of the bay, which was then eaten by
the residents, causing poisoning.
The villagers (8) ________ to the company, but for a long time the company refused to believe
that it was (9) ________. Finally, the villagers were able to receive money from the company and the
government but the (10) ________ effects of the poisoning continue to this day.
1. A. created B. arose C. appeared D. occurred
2. A. destroy B. damage C. ruin D. hurt
3. A. chemical B. biological C. physical D. mathematical
4. A. dis-normal B. counter-normal C. anti-normal D. abnormal
5. A. patients B. participants C. attendants D. pedestrians
6. A. leaving B. pouring C. releasing D. flowing
7. A. affecting B. affected C. effect D. affection
8. A. complained B. begged C. opposed D. worship
9. A. at duty B. at error C. at mistake D. at fault
17
10. A. long-term B. long-run C. long-lasting D. long-time
Passage 3
Many people change countries during their life, but one man has made a (1) ________ for
himself by (2) ________ his own country. During World War II, England built a number of artificial
platforms off the coast of England to contain (3) ________ to fight off the invaders. After the war, the
soldiers left these platforms and they were forgotten until 1967. In that year, a British man, Roy Bates,
(4) ________ one of the platforms and announced he had started his own country, called Sealand.
This (5) ________ as a surprise to England, who believed they had the (6) ________ to the
platform. Bates, who renamed himself Prince Roy, and the small (7) ________ on the island (his
family), (8) ________ a great deal of (9) ________ from England to leave the platform. However, in
1968 an English course decided that because Sealand was over six kilometers off the English coast, it
was in international waters, and England had no power there. Today, Sealand, which even has its own
stamps, coins, and passports, is controlled by Roy’s son and (10) ________, Prince Michael, and is
home to a large Internet business.
1. A. name B. fame C. reputation D. recognition
2. A. creating B. establishing C. working D. making
3. A. plan B. force C. army D. weapons
4. A. took back B. took care C. took over D. took up
5. A. came B. went C. appeared D. happened
6. A. power B. control C. rights D. duties
7. A. workforce B. people C. tribe D. community
8. A. met B. insisted C. faced D. kept
9. A. pressure B. fighting C. disturbing D. threats
10. A. inheritance B. heir C. heiress D. heirdom

Passage 4
When important events are (......1.......) around the world, most people turn to traditional media
sources, such as CNN or BBC, (......2.......) their news. However, during the invasion of Iraq by the
United States and (......3.......) allies in early 2003, a significant number of people followed the war
from the point of view of an anonymous Iraqi citizen who call himself “Salam Pax” (salam means
“peace” in Arabic, and pax mean “peace” in Latin).
Salam Pax wrote a diary about everyday life in Baghdad (......4.......) the war, and posted it in his
website. Pax’s online diary was a site of website known as a “blog”. Blogs, short for “web-logs”, are
online diaries, usually kept by (......5.......), but sometimes by companies and other groups of people.
They are the fastest growing type of website on the Internet. In 2003, there were (......6.......) to be
several hundred blogs on the Internet, and the number was growing by tens of thousands a month.
A blog differs (......7.......) a traditional website in several ways. Most (......8.......), it is updated
much more regularly. Many blogs are updated every day, and some are updated several times a day.
Also, most blogs use special software or web sites which are (......9.......) aimed at bloggers, so you
don’t need to be a computer expert to create your own blog. This means that ordinary people who may
find computers difficult to use can easily (......10.......) and start writing their own blog.

1. A. taking B. happening C. doing D. playing


2. A. for B. by C. of D. from
3. A. their B. it C. its D. theirs
4. A. during B. in C. after D. before
5. A. people B. individuals C. person D. men
6. A. foreseen B. predicted C. estimated D. expected
7. A. of B. in C. with D. from
8. A. importantly B. unimportant C. importance D. important
9. A. specific B. specify C. specifically D. special
10. A. create up B. make up C. fill up D. set up

Passage 5
18
Many researchers claim to have found the ......(1)...... between left-handedness and various ......
(2)...... and mental characteristics, such as blonde hair, blue eyes, vegetarianism, and sleep difficulties.
Other studies have found a higher-than-normal level of left-handed people in ......(3)...... occupations,
including professional baseball and tennis players, architects, lawyers, as well as prisoners. ......(4)......,
some of these connections are very weak, and others haven’t been proved.
What makes a person become right-handed ......(5)...... than left-handed? As yet, no one really
knows for sure. One simple idea ......(6)...... that people normally get right-handedness from their
parents. Studies have found that two right-handed parents only have 9.5 percent ......(7)...... of having a
left-handed child, whereas two left-handed parents have a 26 percent chance of having a left-handed
child. Another common theory is that left-handed people suffer mild brain damage during birth, ......
(8)...... makes them left-handed. However, if this theory were true, it wouldn’t explain ......(9)...... the
percentage of left-handed people is so similar in every society, when birth ......(10)...... vary so much
from society to society.

1. A. relations B. relatives C. relationships D. relate


2. A. hard B. physical C. fixed D. apparent
3. A. certain B. sure C. an D. all
4. A. However B. Moreover C. As a result D. Also
5. A. more B. less C. quite D. rather
6. A. thinks B. suggests C. believes D. says
7. A. chance B. ability C. capability D. step
8. A. that B. who C. which D. whom
9. A. what B. when C. how D. why
10. A. situations B. standards C. requirements D. conditions

Passage 6
About one person ……(1)…… ten is left-handed, but right-handed people tend to overlook the
problems “lefties” face. For example, right-handed people ......(2)...... for granted many simple
everyday tasks such as using scissors, computer keyboards, and driving a car. These ......(3)...... are so
natural that right-handed people don’t even think about them. They certainly don’t take into account
the fact that all these things are designed ......(4)...... right-handed ways of doing things. There are big
problems for left-handed people at work ......(5)...... at home. Many machines can be ......(6)...... safely
only with the right hand. Perhaps this ......(7)...... right-handed people an advantage in being hired for
some occupations. ......(8)......, there’s the problem of discrimination. In the past, some people ......
(9)...... left-handed people. These days, attitudes have changed, but the design problems of inventing
products that can be used with either hand still have not been ......(10).......

1. A. on B. of C. in D. from
2. A. have B. take C. give D. send
3. A. activities B. behaviors C. images D. thoughts
4. A. to benefit B. benefits C. beneficial D. benefiting
5. A. but B. except C. including D. as well as
6. A. worked B. operated C. run D. taken
7. A. gives B. takes C. has D. provides
8. A. However B. Consequently C. In addition D. In conclusion
9. A. looked into B. looked after C. looked up D. looked down on
10. A. located B. cleaned C. raised D. solved

Passage 7
However objective we believe ourselves to be, most of us do not judge a product solely on its
merits, considering quality, value and style before making a decision. ……(1)……, we are easily
influenced by the people around us.
There is nothing ……(2)…… with this. It is probably a smarter way to make decisions than
……(3)…… on only our own opinions. But it does make life hard for companies. They have long
19
understood that groups of friends and relatives tend to buy the same products, but understanding the
reasons has been tricky. It is because they are so similar with ……(4)…… to how much money they
make and what television ads they watch that they independently ……(5)…… at the same decision?
Or do they copy one another, perhaps ……(6)…… envy or perhaps because they have shared
information about the products?
Research in Finland recently found overwhelming evidence that neighbours have a big
influence on buying decisions. When one of a person’s ten nearest neighbours bought a car, the
chances that that person would buy a car of the same brand during the next week and a half ……(7)
…… by 86 per cent. The researchers argued that it was not just a ……(8)…… of envy. Used cars
seemed to attract neighbours even more than new cars. This suggested that people were not trying to
……(9)…… up with their neighbours, they were keen to learn from them. Since used cars are less
reliable, a recommendation of one can ……(10)…… influence a buying decision.

1. A. What’s more B. Instead C. Unlike D. In place


2. A. wrong B. silly C. bad D. daft
3. A. basing B. trusting C. supposing D. relying
4. A. connection B. regard C. relation D. concern
5. A. reach B. come C. arrive D. get
6. A. for B. as to C. out of D. about
7. A. boosted B. rose C. enlarged D. lifted
8. A. thing B. point C. matter D. fact
9. A. keep B. stay C. hold D. follow
10. A. fiercely B. strongly C. firmly D. intensely

Passage 8
If you want to prepare yourself for great achievement and have more to (1) ______ to your
education or your work, try reading more books. (2) ______ up some of the interestingly informative
books and search for well-researched materials that can help you grow.
We should (3) ______ our children to read more books and (4) ______ less time watching TV.
Some people have commented that this is inconsistent. "Why is the written word a superior way to get
information than television?" That is (5) ______ interesting point of view worth further (6) ______.
Reading is a skill that is in much greater demand than the demand for watching TV. There are no jobs
that (7) ______ a person to be able to watch TV but reading is an integral part of many jobs. The
written word is an incredibly flexible and efficient way of communication. You can write something
down and, in no time, it can be (8) ______ to many different people. Not only that, we can (9) ______
vast amounts of information through reading in a very short time. A good reader can acquire more
information in reading for two hours than someone watching TV can acquire in a full day. You are
able to gain a lot of information quickly because you are a fast reader with good (10) ______ skills. It
will save you massive amounts of time and you will be able to assimilate vast quantities of
information.

1. A. gain B. gather C. collect D. contribute


2. A. Make B. Pick C. Set D. Take
3. A. discuss B. define C. suggest D. encourage
4. A. apply B. spend C. train D. waste
5. A. a B. an C. the D. ---
6. A. exploration B. explore C. explorer D. explorative
7. A. require B. inquire C. tell D. ask
8. A. submitted B. communicated C. handed D. sent
9. A. digest B. inhale C. breathe D. eat
10. A. apprehension B. enjoyment C. entertainment D. comprehension

Passage 9

20
Any change in one part of an ecosystem can cause changes in other parts. Droughts, storms and
fires can change ecosystems. Some changes ……(1)…… ecosystems. If there is too ……(2)……
rainfall, plants will not have enough water to live. If a kind of plant dies off, the animals that ……(3)
…… it may also die or move away. Some changes are good for ecosystems. Some pine forests need
……(4)…… for the pine trees to reproduce. The seeds are sealed inside pinecones. Heat from a forest
fire melts the seal and lets the seeds ……(5)…… . Polluting the air, soil, and water can harm
ecosystems. Building ……(6)…… on rivers for electric power and irrigation can harm ecosystems
……(7)…… the rivers. Bulldozing wetlands and cutting down ……(8)…… destroy ecosystems.
Ecologists are working with companies and governments to find better ways of ……(9)……
fish, cutting down trees, and building dams. They are looking for ways to get food, lumber, and other
products for people ……(10)…… causing harm to ecosystems.

1. A harms B. harmful C. harmless D. harm


2. A. little B. a little C. few D. a few
3. A. fed B. feed C. feed on D. food
4. A. flame B. fires C. blaze D. burning
5. A. out B. in C. go D. fly
6. A. moats B. ditches C. bridges D. dams
7. A. on B. around C. over D. under
8. A. hills B. jungles C. forests D. woods
9. A. catching B. holding C. carrying D. taking
10. A. avoid B. without C. not D. no

Passage 10
Set in the red desert of central Australia is the mining town of Coober Pedy. At first sight, the
town looks similar to many other such communities, but Coober Pedy is different. Sixty per cent of its
population of some 4,000 people live underground. There are today about 800 underground houses as
well as shops, hotels and even churches in the town and the ……(1)…… hills. Once a site has been
chosen, special tunneling machines are ……(2)…… in to create passages and rooms in the sandstone.
Rock pillars are left to ……(3)…… the roof, and doors and windows are cut into the front. Houses are
of all shapes and ……(4)……, the largest having twenty rooms, and some even have their own
swimming pool.
Living underground may sound strange but in fact it has a number of advantages. In summer,
the temperature outside can reach an astonishing 47 0C, and in winter the nights can be ……(5)……
cold. However, inside the houses it remains a steady 25 0C all year ……(6)…… . Many people say that
living underground makes them feel very secure. There is no problem with noise from the neighbours
and the houses are not ……(7)…… by the fierce dust storms that regularly ……(8)…… through the
area. And of course, if your family ……(9)…… or lots of friends come to stay, you can ……(10)……
dig another room.

1. A. near B. enclosing C. close D. surrounding


2. A. brought B. entered C. placed D. worked
3. A. lift B. support C. push D. rise
4. A. sizes B. areas C. volumes D. numbers
5. A. strongly B. heavily C. extremely D. sharply
6. A. along B. round C. across D. wide
7. A. spoiled B. disturbed C. interrupted D. affected
8. A. pour B. sweep C. flood D. hurry
9. A. grows B. rises C. stretches D. explodes
10 A. only B. always C. regularly D. ever

21
Part II: Read the passage below and choose the correct answer for each question.

Passage 1
Cairo, May 25, 1998 After ten years of work hard by archaeologists, most of the restoration
work on the Sphinx has been completed. A big celebration was held today to mark the achievement.
Important people from many nations attended, in addition to thousands of Egyptians. For Egyptians,
the Sphinx is more than a monument, it is a visible symbol of their culture. They also hope that the
multi- million- dollar project will bring tourists to benefit the economy.
In an interview with this newspaper, Dr. Zahi Hawass, the archaeologist running the project, said that
the restoration continues to this day. Dr. Hawass said that the Sphinx had been repaired before, but that
some parts of it continue to have problems. Earlier workers used artificial material from which the
monument is made. In addition, the air pollution in the local environment has caused other problems.
Dr. Hawass said that because it is located in the desert, sand had covered the Sphinx many
times, and he said it stands a chance of being covered by sand again. Hawass also said that although
the damage to the head of the Sphinx happened a long time ago, the restoration did not attempt to fix
the nose and beard. Some people were worried that the restoration work would change the Sphinx too
much, so Hawass said the restoration team kept their word to leave the face as it was.
1. When did the restoration work start?
A. in 1998 B. 1988 C. 1990 D. 1980
2. What do Egyptian people think of the Sphinx?
A. It’s a statue. B. It’s a masterpiece.
C. It’s a cultural symbol. D. It’s a place to worship.
3. What causes the problems to the Sphinx?
A. The air pollution B. The materials used before
C. The earlier workers D. A and B are correct
4. What is likely to happen to the Sphinx?
A. To be covered by sand B. To be destroyed by pollution
C. To be removed from the desert D. To be rebuilt in the desert
5. What can be inferred from the text?
A. The restoration team will fix the nose and the beard of the Sphinx.
B. The restoration team promised to keep the original face of the Sphinx.
C. Egyptians people are trying to protest the restoration of the Sphinx.
D. Egyptians people will help the restoration team to do the work.

Passage 2
In many parts of the world, people tell folk stories about magic spiders. The Anansi stories
native to the Ashanti tribe in Ghana are the most famous of these stories. Two hundreds years ago
when people travelled from Ghana in West Africa to the New World, they took the Anansi stories with
them. Today they are very popular in the Caribbean, especially in Jamaica. They are also popular in
both North and South America.
The Anansi stories are about a trickster spider who told a lot of stories. In folk tales, a trickster
is a small but very clever animal who, as the name suggests, plays jokes or tricks on larger, more
powerful animals. Tricksters use their intelligence and their humor to get what they want. According to
the legend, Anansi the spider earned his stories from the sky god Nyame by catching a leopard, some
dangerous hornets (insects similar to bees), and a python (a kind of huge snake). Normally, a spider
wouldn’t stand a chance against these animals, but Anansi figured out how to trap them.
Animal trickster stories have been passed on for generations by older people in a group telling
them out loud to younger people. The stories help people understand each other, and they teach correct
way to act towards others. People all of ages like to listen to them because they are funny and they like
the way the small spider is the hero of the stories. Sometimes, the main trickster animal is the rabbit or
a turtle instead of a spider. The trickster gets into all kinds of difficult situations, but he always
succeeds in the end.
1. Where did the Anansi stories first come from?
A. Africa B. Asia C. Europe D. America
22
2. How can you describe the tricksters?
A. Tall and humorous B. Small and smart C. Big and dangerous D. Big and stupid
3. What species are mentioned in the text as being the tricksters?
A. Spiders B. Rabbits
C. Turtles D. All answers are correct.
4. Why is the meaning of the trickster stories?
A. They teach language of the tribe.
B. They teach people to behave properly.
C. They help the younger people get closer together.
D. They motivate the young to become heroes.
5. What can be inferred from the text?
A. The tricksters always win.
B. The tricksters run very fast.
C. The end of the stories is always unexpectedly.
D. The main characters of the stories come from the same species.

Passage 3
Not many people get the chance to ride in the Tour de France, but people all over the world
ride bicycles. Bicycles are so common that we often take them for granted. In the Netherlands, people
of all ages and lifestyles ride bicycles instead of using other types of transportation. Along any bike
path, you’ll see businessmen with their briefcases, mothers with small babies, and elderly
grandmothers cycling home after food shopping. In Southern Asia, goods are often transported from
place to place on bicycles. High piles of boxes are taken by bicycle from one town to a nearby one. In
India and Indonesia, bicycle rickshaws are used instead of taxis. It is cheaper to use bicycles than other
forms of transportation, and it’s better for the environment, too, since bicycles create no air pollution.
In other countries, people ride bicycles mostly for recreation as a hobby or sport. Recreational
cycling is made up of several different types of activities. Cycle touring involves riding bicycles for
long distances, often in places with spectacular scenery. Touring cyclists carry everything they need
with them in small bags called panniers that hang from their bicycles. They take their rout into account
and bring equipment, clothing, and everything needed for repairs if they are planning to travel far from
towns. They can’t depend on finding a garage or repair shop to fix problems. Although they travel with
all their equipment, their bicycles are quite light so they don’t have to use a lot of energy to ride them.
1. Who may ride bicycles in the Netherlands?
A. Elderly people B. Teenagers
C. People aged from 30-55 D. All answers are correct.
2. Why do people choose to ride bicycles?
A. Because it’s better for health.
B. Because it’s environmentally friendly.
C. Because it’s easier to buy.
D. Because it’s safer for riders.
3. Where do the touring cyclists often ride to?
A. Dangerous places B. Rural areas
C. Places with beautiful scenery D. Places near their home4. What may not
be found out in the panniers?
A. Clothes B. Dolls C. Fixing tools D. Equipment
5. What can be inferred from the text?
A. Cyclists do not feel too tired during the trip. B. Cyclists feel exhausted during the trip.
C. The bikes are big and heavy. D. The bikes are modern.

Passage 4
Cosmetic surgery gets more popular all the time. A recent survey by a British television station
found that 66 percent of British people want to change the way they look. According to the survey, one
tenth have already had plastic surgery. It’s popular in America, too, where many people want to fix
features they think are ugly. However, several people have died recently while having these kinds of
23
procedures. A well-known author from California made the headlines in 2004 when she died while
having wrinkles on her neck removed. Medical specialists point out that it’s very important to check to
see that your doctor and the institute you choose have a good reputation.
In Taipei, many people schedule cosmetic surgery right before Chinese New Year. For one
thing, they want to start a new fresh in the new year. Also, they want to take advantage of the holiday
to recover from the procedure. Soon after the holiday period, any scars from the surgery have
disappeared.
Some patients use an alternative to plastic surgery. They prefer injection of Botox that rapidly
makes wrinkles disappear. To an extent, this procedure pays off immediately with visible results.
However, the drawback is that the injections are not permanent. They enable people to look different
for a few months. Then they can expect to return and have the injections again.
1. What percentage of British people are happy with their looks?
A. 66% B. 34% C. 10% D. 90%
2. What did the victim who died in 2004 do?
A. A teacher B. An actress C. A doctor D. A writer
3. Why do people schedule cosmetic surgery before the New Year holiday?
A. Because they have more money B. Because it’s cheaper.
C. Because doctors are less busy. D. Because they are off work.
4. What is the problem with injections?
A. Injections are temporary. B. Injections are more dangerous.
C. Injections are more expensive. D. Injections last forever.
5. What can be inferred from the text?
A. Cosmetic surgery is just for famous people.
B. People with more money will choose injections.
C. More people get cosmetic surgery in Britain than in the US.
D. It’s easy to recognize the results of injections.

Passage 5
One of the worst oil spills occurred in 1978 in northwestern France. A ship named the Amoco
Cadiz was travelling from the Arabian Gulf to a French port when it got into trouble. It ran aground in
shallow water and then broke apart. At first, it was not clear who was at fault because the accident
happened in a bad storm. Later, people learned that the steering equipment on the ship was broken.
High winds and tides were the main factors that caused the disaster. About 220,000 tons of oil
rapidly spilled out of the damaged ship. It floated to shore and covered everything. Within a month, the
oil covered over three hundred kilometers of neighboring coastline.
The oil affected the environment badly. It harmed many species and animals, fish, and plants. It
caused long-term damage to the local economy that relied on fishing and tourism. To an extent,
scientists were able to use detergents to clear up the oil, but then they found that detergents damaged
the environment too. Some of the oil continued to cause problems for three years.
1. The Amoco Cadiz oil spill occurred because of
A. the storm B. the captain C. broken equipment D. shallow water
2. What made the oil spill become the disaster?
A. High winds and tides B. Sea water
C. Another ship D. Steering equipment
3. How long did it take the oil to cover the coast for more than 300 kilometers?
A. More than a month B. Within 30 days
C. Half a month D. Half a year
4. What are the most significant industries of the local area?
A. Fishing B. Ship making
C. Tourism D. A and C are correct
5. What can be inferred from the text?
A. Oil spills affect both natural life and human life.
B. Oil spills often occur in the spring.
C. Causes of oil spills are difficult to be found out.
24
D. Scientists cannot do anything to clear up the oil.
Passage 6
In the mid-1800s, there was a serious problem getting mail from the eastern part of the US to
California on the Pacific Coast. Many settlers had moved westward in the first half of the nineteenth
century, but they wanted to stay in contact with families and friends back in the East. They didn’t like
waiting for six weeks or longer to get a letter. People in California felt they couldn’t rely on the regular
postal service.
The Gold Rush in 1848 brought even more people to California. At that time, most mail went
by sea from New York to Panama in Central America. There it was taken by railroad across the narrow
strip of land that separates the Atlantic and Pacific Oceans. Once it reached the Pacific side, it was
transferred to a ship to travel north to California. This took a long time too.
When America tried to overcome its communication problem in the 1860s, the nation had
many historical models to take into account. In thirteenth century China, the Mongol Empire
established communication routes across central Asia. Couriers rode horses between a series of rest
stations. The couriers rode rapidly until the next stop where a fresh rider was waiting to take over.
In the fifteenth century, the Incas had a similar system in South America. The Incas built
16,000 kilometers of roads and a message could travel 400 kilometers in one day.
1. How long did people in California have to wait to get a letter from the eastern part of the US?
A. A week or more B. A month and a half or more
C. Two months or more D. More than a month
2. How were letters transferred from New York to Panama in the mid-1800s?
A. By wagon B. By train C. By ship D. By plane
3. What happened to the mail when it reached Panama?
A. It was delivered to recipients. B. It was kept there.
C. It was sent back to New York. D. It was taken by railroad to the Pacific side.
4. How did the couriers travel in the Mongol Empire time?
A. On foot B. By horse C. By ship D. By train
5. What can be inferred from the text?
A. America learned something from the communication systems of other countries.
B. America invented the modern communication system.
C. In the past, all letters were delivered by sea.
D. In 19th century, China built roads in Asia.

Passage 7
People use more than just words to communicate. In fact, some researchers claim that less than
half of a spoken message’s real meaning is in the words used in the message. They say that most of the
message’s meaning comes from understanding how the speaker uses things like tone of voice and body
language.
Body language includes such things as the expression on the speaker’s face, gestures the
speaker makes with his or her hands, and the position of the speaker’s body. Just as there are many
different languages spoken around the world, there are many different ways for people to use body
language, too. For example, gestures may imply different meaning in different cultures. Making a
“thumbs up” sign in America means “Great”. However, in Arab culture, that gesture is extremely
offensive. And in India, if listeners want to show speakers that they understand, the listeners will move
their heads from side to side. In many western countries, a similar gesture means “no”.
Although many gestures can be interpreted differently by different cultures, there are also many
gestures that are almost universally interpreted the same way. For example, by and large a smile is
understood as a sign of friendship or good will around the world. Also using an open hand to gesture
towards something is viewed as polite or friendly in most cultures.
1. According to the text, what should be based on to understand a message besides words?
A. Nothing else B. Voice tone and body language
C. Stressed words D. Speakers’ appearance
2. What do languages and body language have in common?
A. They are both diverse. B. They are both spoken in the world.
25
C. They are both easy to interpret. D. They are both written down.
3. What can be based on to understand the meaning of a gesture?
A. The way it is done B. The culture it comes from
C. The person using it D. The situation it is used
4. What do the people in western countries mean if they move their head from side to side?
A. They agree B. They are angry C. They mean “yes” D. They mean “no”
5. What can be inferred from the text?
A. A smile implies a positive message.
B. Body language can replace languages.
C. Body language is only useful to the disabled.
D. People should not use gestures in formal situations.

Passage 8
Welcome to the world of speed dating! This is the latest way for single people to meet other
people they might like to meet without the pressure associated with the conventional dating scene.
Many businesses offering speed dating service have become especially popular in the US,
England, and Canada. These businesses organize events where participants can have dozens of quick
dates all on the same night. Each speed date is less than ten minutes, and some services limit each date
to only 3 minutes. This gives people just enough time to make an impression, so that each person in
the date can decide if they want to meet the other again. If both people mark “Yes” on the date card,
the service will arrange for the people to contact one another to arrange a real date.
The idea for speed dating came from Yaacov Deyo of Los Angeles. In 1999, he wanted to
provide an alternative way for his students to get together other than using blind dates arranged
through families or friends. His speed dating idea was so successful that it quickly spread to other
communities across the US. Deyo and his wife also wrote a book, Speed Dating: A Timesaving Guide
to Finding Your Lifelong Love, to teach other people the keys to successful speed dating. Then
businesses were set up to try to cash in on the popularity of speed dating. And after a popular
television sit-com did an episode on speed dating, dating services across the US really took off.
1. Why does speed dating service become popular?
A. Because it does not involve pressure.
B. Because it is more open.
C. Because it is less expensive.
D. Because it takes less energy to conduct.
2. How long might it take a person to complete ten speed dates?
A. ten minutes B. one week C. one evening D. ten days
3. Who often arranges a blind date?
A. Companies B. Parents C. Teachers D. Daters
4. Who is Deyo?
A. A man who has had many speed dates.
B. A man who got married after speed dating.
C. The inventor of speed dating.
D. The publication of a book in Milwaukee.
5. What can be inferred from the text?
A. Speed dating is the guarantee for a wedding.
B. All young people should go to the speed dates.
C. All speed dates work out.
D. People do not have much time to know each other well in a speed date.

Passage 9
Wikipedia is an encyclopedia that is available on the Internet and what people love about it is
that it can be edited by absolutely everybody.
When and how did it start?
It was founded in 2001 by a guy called Jimmy Wales. It started as a fancy idea, a kind of a
hobby and everybody is surprised how popular it has become and how many computer scientists it has
26
attracted. It has got a collection of about 1.8 million articles, the majority of which are in English;
however, one can find some articles in over 200 languages. If it was a business, it would earn lots of
money.
How is it possible that articles that can be changed by anyone are correct?
The Wikipedia is based on wikis - a special software which lets everyone modify a webpage
and it is true that anyone can change the information on the page if they think it's incorrect. But, the
Wikipedia has a team of over 13,000 people who are experts in different fields and who correct any
inaccurate information sent by people.
Is it error-free?
One may say so. Recently, for example, the British journal Nature looked at the scientific
information in Wikipedia and confirmed it was very reliable and that they didn't find many errors. It
was very good news for the founder as well as for all the users.
Why is it becoming so popular?
Like the whole idea of the Internet, it's also quick and available to everyone. The greatest thing
of all is that it is free. Some people also stress that it's fun to be able to add what you know to the
information on the net. IT specialists believe it has a very bright future and most claim it's the most
brilliant invention ever.
1. Wikipedia ………
A. was created by a team of computer scientists.
B. began as a business idea.
C. became popular as soon as it started.
D. started as one man's passion.
2. Articles in Wikipedia are ………
A. mostly about science. B. mostly in English.
C. translated into 200 languages. D. very interesting.
3. Wikipedia remains accurate as much as possible because ………
A. all people who write for it are experts.
B. it has a special type of software programme.
C. there are people who monitor it for mistakes.
D. not everybody can change the information.
4. The best advantage of Wikipedia is that ………
A. you don't have to pay for it. B. everyone can use it.
C. it is created by ordinary people. D. is quick and reliable.
5. The text probably comes from ………
A. a leaflet B. a speech
C. a scientific article D. a magazine article

Passage 10
In today's competitive world, what responsible parent would not want to give their children the
best possible start in life? For this reason, many parents want their children, often as young as ten
months old, to become familiar with computers. They seem to think that if their children grow up with
computers, they will be better equipped to face the challenges of the future.
No one has proved that computers make children more creative or more intelligent. The truth
may even be the opposite. Educational psychologists claim that too much exposure to computers,
especially for the very young, may negatively affect normal brain development. Children gain valuable
experience of the world from their interaction with physical objects. Ten-month-old babies may benefit
more from bumping their heads or putting various objects in their mouths than they will from staring at
eye-catching cartoons. A four-year-old child can improve hand-eye coordination and understand cause
and effect better by experimenting with a crayon than by moving a cursor around a computer screen.
So, as educational psychologists suggest, instead of government funding going to more and more
computer classes, it might be better to devote resources to music and art programs.
It is ludicrous to think that children will fall behind if they are not exposed to computers from
an early age. Time is too precious to spend with a "mouse". Now is the time when they should be out
there learning to ride a bike. There will be time later on for them to start banging away at keyboards.
27
1. Why do parents want their children to learn how to use a computer from an early age?
A. Because they are afraid their children will become competitive.
B. Because they want their children to be well prepared for their future.
C. Because this is what all the other parents seem to do.
D. Because they believe their children will have difficulty learning to use one if they don't start
early.
2. Children who spend a lot of time on their computers ……...
A. do not necessarily make more progress than those who don't.
B. tend to like music and art more than those who don't.
C. will suffer from brain damage.
D. tend to have more accidents than those who don't.
3. The author implies that children learn better ………
A. after they have developed hand-eye coordination.
B. when they use a computer.
C. as they get older.
D. when they hold and feel things around them.
4. What would be an appropriate title for this passage?
A. Never too early to start B. Let kids be kids
C. Computers in schools D. More computers mean brighter future
5. What is true according to the passage?
A. It is better for children to take computer lessons than art lessons.
B. Parents should not put off buying a computer for their children.
C. Computers seriously harm children's eyesight.
D. There is no evidence that children who use computers are cleverer than those who don’t.

Passage 11
A massage is relaxing, and makes you feel great, but did you know that it's also good for you?
That's what doctors are now saying. Massage relieves pain and anxiety, eases depression and speeds up
recovery from medical problems.
Research has shown that people of all ages benefit from touch. Premature infants who are held
develop faster than those left alone, and healthy babies who get a lot of physical contact cry less and
sleep better. Researchers are not sure why this occurs but they have also found out that touch can slow
heart rate, lower blood pressure and increase levels of serotonin, the brain chemical that is linked to
well-being. It also decreases levels of the stress hormone cortisol, and this in turn increases your
resistance to illness.
Massage also speeds up healing. Bone-marrow transplant patients who were given massages
had better neurological function than those who weren't. Furthermore, massage reduced pain by 37%
in patients with chronic muscle aches.
Giving someone a massage may be as good as getting one. A study conducted by the university
of Miami found that mothers suffering from depression felt better after massaging their infants. In that
same study, elderly volunteers who massaged infants reported feeling less anxious and depressed.
It even works when you do it yourself; 43% of headache sufferers reported getting relief after
massaging their temples and neck and smokers who were taught self-massage while trying to quit felt
less anxiety and smoked less.
1. What has recently been said about getting a massage?
A. It relaxes you. B. It makes you feel good.
C. It improves your physical condition. D. It requires a special technique.
2. Babies born before their time ______
A. cry less and sleep better if they are massaged.
B. grow faster if they are held.
C. develop faster than healthy babies if they get a lot of physical contact.
D. don't survive if they are not held.
3. The author suggests that touch ______
A. increases levels of the stress hormone cortisol.
28
B. makes your heart beat faster.
C. increases the feeling of well-being.
D. helps you deal with your feelings.
4. Patients who get massages ______
A. don't experience muscle pain. B. avoid having surgery.
C. make a quicker recovery. D. are not better off than those who do not.
5. According to the article ______
A. massage has no effect on smokers.
B. massage relieves headaches by 43%.
C. smokers who gave others massages felt less anxious and smoked less.
D. massaging yourself is as effective as being massaged.

Passage 12
If we took a look at how people in Europe communicated just one hundred years ago, we
would be very surprised to find out that English was hardly used outside the United Kingdom. The
language most commonly used between people of different nationalities, and particularly the
aristocracy, was French. In fact, French was the language of diplomacy, culture and education.
However, that is not the case nowadays. English has replaced French as the international language of
communication. Today there are more people who speak English as a second language than people
who speak it as a first language.
There are many reasons why English has become the language of international communication.
Britain's colonization of many parts of the world had something to do with it, but it is mainly due to
America's rise to the position of major world power. This helped spread popular American culture
throughout the world bringing the language with it.
But is it good that English has spread to all parts of the world so quickly? Language specialists
seem to be divided over this issue. There are those who claim that it is important to have a language
that the people in our increasingly globalized world have in common. According to others, English is
associated with a particular culture and therefore promotes that culture at the expense of others.
Linguists have suggested "Esperanto", an artificially put-together language, as a solution to
international communication problems but without success. So, English will continue being the world
language until some other language, maybe Chinese, which is the most widely-spoken native language
in the world, takes over as the world's international language instead of English.
1. According to the passage, a century ago ______
A. educated people throughout Europe spoke English.
B. foreign travelers to England spoke only French.
C. French was much more popular than English.
D. only the French aristocracy could speak English.
2. What is chiefly responsible for the growth in popularity of English?
A. Britain's becoming an international power. B. The French losing many colonies.
C. America's becoming powerful. D. The development of American culture.
3. What is true according to the passage?
A. The experts don't like Esperanto.
B. Esperanto is difficult to learn.
C. Esperanto is not a natural language.
D. Esperanto is becoming more and more popular.
4. These days ______
A. French is the language of diplomacy.
B. more non-natives speak English than natives.
C. more people speak French than English.
D. French is a dying language.
5. What would be a good title for this passage?
A. English; Past, Present and Future B. English as an international language
C. English language means English culture D. English: a difficult language to learn

29
Passage 13
The term "dyslexia" is used to describe a number of problems associated with reading, writing
or spelling. Short-term memory, mathematics, concentration, personal organization and sequencing
may also be affected. We do not know exactly what causes dyslexia, but we do know that it tends to
run in the family. We also know that more boys suffer from dyslexia than girls and that dyslexia is
more common in urban areas than in rural.
One of the most common signs of dyslexia is "reversals". People with this kind of problem
often confuse letters like "b" and "d" when reading and writing or they sometimes read and write
words like "tip" and "won" as "pit" and "now". Other common characteristics are lack of punctuation,
misspelling, mixed-up sentence structure and poor grammatical construction.
People are born with dyslexia, but it is only when they begin to learn to write that it becomes a
noticeable problem. For children with dyslexia, going to school can be a traumatic experience. Poor
achievement can make them feel frustrated and insecure. They are reluctant to go to school and
sometimes even skip school altogether. Cheating, stealing and experimenting with drugs can also
occur when children regard themselves as failures.
It is a common misconception that dyslexic people are of inferior intelligence. Yet Albert
Einstein, Leonardo da Vinci, Thomas Edison and Hans Christian Andersen were apparently all
dyslexic. There is no total cure for dyslexia; however, the effects of dyslexia can be lessened with the
guidance of skilled specialists and a lot of determination.
1. When does it become apparent that a child may be suffering from dyslexia?
A. when they are born B. when they begin their education
C. when they use symbols instead of words D. when they start talking
2. According to the passage, dyslexic children ______
A. might have behavioral problems. B. end up taking drugs.
C. look forward to going to school. D. do well at school.
3. The passage states that one common characteristic of dyslexics is that they ______
A. have difficulty expressing their feelings. B. are not able to read three-letter words.
C. have difficulty learning the letter "b". D. read words backwards.
4. The purpose of this passage is to inform readers that
A. Albert Einstein was dyslexic.
B. it is difficult for dyslexics to get professional help.
C. dyslexics are as intelligent as other people.
D. dyslexics are inferior to other people.
5. How can the effects of dyslexia be made less severe?
A. by learning to live with them B. by learning a special skill
C. with professional help and hard work D. by learning how to deal with failure

Passage 14
Tennis started in France nearly 1,000 years ago. The game was originally played in the
courtyards of royal palaces, using the walls (like squash) rather than a net. One of the Grand Slam
tournaments takes place in Wimbledon every year. The Tournament or The Fortnight, as the British
call the Wimbledon Tennis Championships, is very important to the English.
So here you are, standing in the queue to buy your ticket to watch the matches. Everybody is
waiting for their turn to get inside. Nobody is pushing. If you are English, you will have all the
necessary things with you: a thermos of tea (of course), a folding chair and (surprise, surprise) an
umbrella!
At last you go through the gates, and you discover the atmosphere. People are sitting under
their umbrellas enjoying the British weather. The atmosphere is calm and controlled. You feel as if you
are in a select private club - and, in fact, you are. The gardens are superb (well, you are in England...).
Every year 3,500 geraniums are planted!
What is so special about Wimbledon? Well, it is the oldest tournament in the world, and the
last of the big four championships to be played on natural grass. The American, Australian and French
Championships are played on cement, artificial grass and clay. All the players must dress only in
white. Wimbledon is free from sponsorship, which makes it different from almost all other sporting
30
events. This means that there are no advertising banners around the courts. The people who come to
watch the matches, compared to those who watch many other international tournaments, are well
disciplined. You can only sometimes hear shouts or whistles when a player prepares to serve. And if
any spectator behaves badly, he or she may be asked to leave.
You think that the English are very serious tennis fans. But if you want a good place, you may
well find one around 4 p.m. Where has everybody gone? Look in the tents: they are having
strawberries and tea. After all, tennis is just one of many traditions, and the English like to continue
them all - especially tea!
1. The Fortnight is ______
A. another name for the Grand Slam tournaments.
B. the original name for a game similar to squash.
C. the name of one of the two Wimbledon tournaments.
D. another name for the Wimbledon Championships.
2. To enter Wimbledon you have to ______
A. wait in a line of people. B. book the tickets earlier.
C. have your own chair. D. belong to the club.
3. Which of these sentences is true?
A. Most of the courts at Wimbledon have artificial grass.
B. Only the Wimbledon tournament is played on natural grass.
C. The Wimbledon championships are played on cement or clay.
D. All four Grand Slam Tournaments are played on natural grass.
4. Spectators at Wimbledon ______
A. never behave badly during a match. B. leave when a player serves badly.
C. do not often shout during a match. D. are given special discipline rules.
5. The text is mainly about ….
A. the most popular sports in England. B. the history of tennis championships.
C. the tennis tournaments at Wimbledon. D. different English customs and traditions.

Passage 15
Back in 1853, at the age of 24, Levi Strauss opened a west coast branch of his brother's dry
goods business in New York. Over the next twenty years, he built his business into a lucrative
operation.
One of Levi's customers was a tailor by the name of Jacob Davis. Originally from Latvia, Jacob
lived in Reno, Nevada, and regularly purchased bolts of cloth from Levi Strauss & Co. Among Jacob's
customers was a man who kept ripping the pockets on the pants that Jacob made for him. Jacob tried to
find a way to strengthen his customer's pants when, one day, it finally occurred to him. He decided to
put metal rivets on the pocket corners and at the base of the button fly. It worked and the pants became
an instant success.
Jacob knew he had discovered something new and worried that someone might steal his idea.
That's why he decided to apply for a patent, but he didn't have the $68 that was required for the
paperwork. So, he turned to Levi Strauss. He wrote him a letter suggesting that they hold the patent
together. Being the businessman that he was, Strauss agreed immediately seeing the potential for this
new product. So, on May 20, 1873, the two men received patent number 139,121 from the US Patent
and Trademark Office and went into business together. That was the day blue jeans were born. Who
would have thought back then that denim, thread and a little metal would become the most popular
clothing product in the world and it's all thanks to two men - Levi Strauss and Jacob Davis.
1. According to the passage, why did Jacob write to Levi?
A. He didn't have the necessary funds to get a patent.
B. He didn't know how to apply for a patent.
C. He had no one else to turn to.
D. He knew Levi had the right connections.
2. What is true about Jacob Davis?
A. He was Levi's friend. B. He was Levi's customer in Latvia.
C. He was Levi's business partner. D. He was Levi's tailor.
31
3. Why did Jacob put metal rivets on the pants he made?
A. because his customers asked for it
B. because he wanted to prevent something from happening
C. because it was fashionable
D. because he wanted to do something different
4. Why did Levi accept Jacob's offer?
A. because he knew it would pay off
B. because his business was in trouble
C. because he was looking to expand his business
D. because Jacob was a good customer
5. In 1873, Levi and Jacob ______
A. founded their first company. B. applied for a patent number.
C. made their first blue jeans. D. got their first customers.

Passage 16
HEALTHY LIVING FOR TEENAGERS
Food
In a recent government survey on healthy eating teens scored only 5 out of 10 (8 indicated a
healthy diet and 6 a "passable" one). Only 1 in 10 teens eats the recommended amount of fruit and the
only vegetable that many teens eat is "chips". Most teens in the developed world are eating too much
but are still not getting the vital nutrients to help them grow and stay healthy. More information about
nutrition and healthy eating is needed to help young people eat properly. Teens who diet
often cut out food they need, such as bread or milk, because they think it is fattening. Others don't
know what foods to choose in the school canteen in order to have a balanced diet. There is a saying
"you are what you eat". So, if you want to become the next David Beckham then you'd better start
eating properly.

Exercise
Lack of money in schools plus increased pressure to do well in the course exams means that
teenagers are doing less sport in school than ever before. Girls, in particular, are more likely to suffer
from lack of exercise and up to 4 in 10 girls stop playing sports in their early teenage years. Just
because you aren't sporty doesn't mean you can't be active. Walk or cycle to school instead of taking
the bus. Help at home with the housework or gardening. Go dancing with your friends. There are lots
of ways you can stop being a couch potato!

Sleep
If "we are what we eat" then sleep is like food for the brain. Teens need at least 9 hours' sleep
every night and even mild sleepiness can affect your performance, humour and health. Lack of sleep
can make you tired, angry or depressed. Nearly 40% of secondary school students go to bed after 11
p.m. on school nights and 15% of teens say they have fallen asleep during class. In the USA some
schools are starting classes at 10 a.m. so that teens can get some extra sleep. These schools have
noticed an improvement in their students' work.
1. According to the text, teens who go on a diet cut down on ______
A. vegetables. B. bread and milk. C. chips. D. meat.
2. Most teens in the developed world ______
A. eat too much but are not eating healthy food. B. eat properly and stay healthy.
C. eat less and grow and stay healthy. D. eat a lot of vegetables and healthy food.
3. Schools put pressure on students to ______
A. do well in sports.
B. improve their physical condition.
C. do well in course exams.
D. take a bus instead of walking or cycling to school.
4. According to the text, many secondary school students on school nights ______
A. sleep at least 9 hours. B. feel angry or depressed.
32
C. play computer games. D. go to bed after 11 p.m.
5. According to the text, some schools in the USA have changed the starting time of lessons because
______
A. they wanted to please their students.
B. they wanted to improve students' performance.
C. parents insisted on changing the time.
D. teachers complained about how unmotivated the students were.

Passage 17
Singapore is dominated by diverse ethnic, such as Chinese, Indians and Malays. In addition,
there are many foreigners from Europe, for example UK, Germany and Sweden, who work or study in
this country. Since many companies in Singapore have a high demand of professional workers, the
country is attracting more and more foreigners, especially from the U.S., Japan and Australia. Because
there are many immigrants living in the country, it affects the social life in Singapore. This
multicultural condition influences all aspects of life in Singapore. One of the most noticeable things is
the cuisine. A traveller will find many types of food that can be found in all corners of the city. The
local dishes in Singapore are influenced by the diverse cultures of its residents. You can taste various
kinds of dishes, including Indian and Chinese foods. In addition, there are many street vendors as well
as cafes or restaurants selling great food at affordable prices. No wonder there are so many people who
make this country as the destination for their holidays.
Besides the great selection of foods, there are also many events and festivals in Singapore
which describes the cultural wealth in this country. One major event was the celebration of the Chinese
New Year. The Chinese community holds a big event to celebrate this special occasion. Moreover, not
only the Chinese, the other races communities also participate in the event. It is a form of tolerance and
respect between communities.
1. What is the writer's main aim in writing the text?
A. To explain how Singapore is a wonderful destination.
B. To talk about the multicultural society in Singapore.
C. To introduce the festivals in Singapore.
D. To talk about Singaporean food.
2. Which country is NOT mentioned as part of community in Singapore?
A. America B. Japan C. South Africa D. China
3. What aspect of life is influenced by immigrants?
A. Education B. Culture C. Cuisine D. All of the above
4. What does the writer say about food in Singapore?
A. It is imported from many countries. B. It is sold only in the café or restaurants
C. It is diverse and cheap. D. It is influenced most by Indian and Chinese
immigrants.
5. Why does the author talk about Chinese New Year celebration?
A. To show the harmony among communities. B. To invite readers to enjoy it.
C. To advertise Singapore tourism. D. To describe the big event in Singapore.

Passage 18
Indonesia is a group of islands, located between Asia and Australia. It is close to the equator, so
it never gets cold. Every day, the temperature is between 21°C and 32°C. Indonesia is covered with
tropical rainforests. For many years, Indonesians have used wood to build their homes. Most of the old
houses have wooden frames, walls and floors. Local people have cleared land for farming, and loggers
have cut down miles of trees to sell the wood to other countries. Now there is a serious shortage of
trees.
Nowadays, in Indonesia's rural areas, many of the houses are built on stilts. They raise the
houses up, keep the living area off the wet soil, and also keep the houses from flooding during the
rainy season. Some people still live in houses with only one room, and some of these houses do not
have electricity or running water, but they cannot afford to improve their housing.

33
In Indonesian cities, more and more people live in crowded apartment buildings. However, it is
hard for people to find good housing. The electricity often goes out throughout these whole buildings.
Moreover, landlords do not want to rent their property to poor people, and banks do not want to lend
them money to buy homes.
Because of its location, natural disasters such as floods, droughts, earthquakes and forest fires
frequently occur, which destroys buildings and leaves people homeless. These disasters affect people
both in the city and in the country.
1. According to the passage, what causes the shortage of trees in Indonesia?
A. Clearing land for farming B. Building wooden houses
C. Smuggling wood D. All answers are correct.
2. What does the word “They” in paragraph 2 refer to?
A. A kind of material B. A part of house C. A type of house D. A name of place
3. What natural disaster is NOT mentioned in the passage?
A. Floods B. Droughts C. Storms D. Earthquakes
4. What makes poor people difficult to find houses in cities of Indonesia?
A. Lack of housing B. Lack of financial support from banks
C. Lack of electricity or running water D. Lack of trees
5. What can be the title for the passage?
A. Indonesian houses B. Indonesian culture
C. Indonesian housing problems D. Indonesian lifestyle

Passage 19
Making schoolchildren take tests is harming students' interest in science, as well as having a
negative impact on kids' natural curiosity. This is the conclusion of a British university report into
science teaching and testing in primary schools. Researchers from Durham University warned that too
many schools were teaching science just so students could pass tests. There is a lot of pressure for
students to get to better schools. The experts said there was very little real science teaching going on
encouraging students to find out things for themselves. Learning by doing, experimenting and seeing
should be at the heart of all physics, chemistry and biology lessons. The Durham University team
found the opposite; that there was little hands-on, practical work taking place in Britain's schools.
Lead researcher Professor Peter Tymms said it was important to develop new approaches to
primary school science. He compared today's teaching with that of the past sixty years and found that
testing was harming children's natural desire to ask questions about science: “We suspect that the
current national approach to science in primary schools is not impacting on children's scientific
thought and curiosity as much as possible,” he said. Professor Tymms made it clear what schools and
science teachers need to focus on, saying: “The purpose of science in primary schools should be to
foster a sense of curiosity and positive attitudes in the young child. It should also guide the child in
solving problems to do with the physical, natural and human worlds.”
1. What does the report find out about teaching science in primary schools?
A. Students have to do too much work.
B. Students are encouraged to find out things for themselves.
C. Students now have a lot of pressure and they don't have time to study science.
D. Students are taught to pass the test and that destroys their interest in science.
2. What is a good way of learning and teaching science according to the report?
A. Making students take tests.
B. Put a lot of pressure on them to encourage them to find things for themselves.
C. Learning by doing, experimenting and seeing to inspire students.
D. Teaching by harming students' natural curiosity.
3. What does this passage imply about teaching science in the past?
A. It encouraged students to ask questions about science.
B. It was worse than today's teaching because it harmed students' interest.
C. It had negative impact on students' scientific thought and curiosity.
D. It helped to create new teaching approaches.
4. According to the writer, what is the purpose of science in primary schools?
34
A. To increase kids' curiosity and positive attitudes, and to guide students to solve problems.
B. To focus on testing students' knowledge and get better school.
C. To help students pass the test and to answer the questions teachers give.
D. To solve students' problem with the physical, natural and human worlds.
5. What does the writer recommend about current science teaching?
A. It should be like the teaching approach in the past sixty years.
B. Schools and science teachers should focus on the purpose of science teaching.
C. It should not influence too much on children's scientific thought and curiosity.
D. It should have little hands-on, practical work taking place in schools.

Passage 20
The world business and trade have become very fast, easier and more reliable because of the
development and improvement of technology. You can now do business with a person who is very far
from you without any difficulties. It has become easier to book railway tickets, bus tickets and even
flights at the comfort of your home without having to move. The introduction of ATMs has made it
very possible to withdraw money at any time of the day. Also, it has become easy to advertise through
the internet and thereby promoting the business in a faster way. Banks now fill data in the computer
instead of doing it manually as they used to do in the old days.
This makes it easier to find the name of the customer, address, email address, birth date, phone
numbers and even the location. Also, the clients can get a very quick access at their account balances
by logging in to the bank's website. With the rise of computer technology, credit cards and other
plastic money, it is very useful especially when one does not have cash at hand or when one is ordering
online products. Communication between partners has been made easier, for example, the internet has
brought email and chatting facilities. Instead of sending letters to respective destinations, it is very
easy to send an email to many people at the same time.
1. What is the writer's main aim in writing the text?
A. To explain how technology has greatly improved business and trade.
B. To explain how to do business online.
C. To explain how to advertise through the internet.
D. To explain how to make money transfer.
2. What field is NOT mentioned in the passage?
A. Finance B. Marketing C. Education D. Communication
3. What does the word “manually” refer to?
A. By a machine B. By hand C. By technology D. By electricity
4. What are examples of technology listed in the passage?
A. ATMs B. Computer C. Internet D. All are correct.
5. What does the writer say about email?
A. It can be sent to many destinations. B. It won't be sent to wrong addresses.
C. It is easy to find through computer. D. It is suitable for all ages.

Passage 21
You've probably heard the expression ‘Laughter is the best medicine'. Well, scientists and
medical doctors now agree that laughter can help people deal with stress and anger. Research has also
shown that people who feel happy and relaxed are healthier and have better relationships. Laughter
yoga is a series of exercises which teaches you to laugh for no reason. You don't need to have a sense
of humour, or understand funny jokes. You just have to laugh. When you do laughter yoga in a group,
you laugh and do yoga breathing, so that you increase the amount of oxygen in your body. Soon the
laughter exercises turn into real laughter. After a laughter yoga class, you feel calm and relaxed, but
also happy and full of energy. Laughter yoga was developed by a medical doctor from India, Dr
Madan Kataria. He started the first laughter yoga classes in a park in Mumbai in 1995 with just five
people, and now there are over six thousand laughter clubs in sixty different countries.
Companies who have run laughter yoga workshops find that they have advantages for both
employers and employees. People who are happy at work are more hard working and make more

35
money for the company. Also, people who can laugh together, communicate more successfully. In
general, people do better at work when they feel happy and relaxed.
1. What is the writer's main aim in writing the text?
A. To confirm that laughter is the best medicine B. To introduce laughter yoga
C. To investigate the feeling of people D. To advise people not to laugh too much
2. According to research, which statement below is right?
A. Laughter makes people look younger.
B. Laughter can improve people's health.
C. People will feel angry without laughter yoga.
D. People who don't laugh cannot build relationships.
3. What do people do in the laughter yoga class?
A. Laugh and do yoga breathing B. Tell funny jokes and laugh
C. Do exercise for strictly medical purpose D. Join outdoor activities for charity
4. According to the text, what are benefits of laughter yoga?
A. Feel calm and relaxed B. Feel happy and full of energy
C. Work and communicate more successfully D. All are correct.
5. Today, laughter yoga …
A. is still popular only in India.
B. has spread to different countries all over the world.
C. is popular only among young people.
D. has become a favourite pastime of many company directors.

Passage 22
I had been happily working in the same company as a graphics designer for over ten years, so it
came as a huge surprise when one day, without warning I was made redundant. In fact, most of the
people in my department lost their jobs that day. When the personnel manager gave me the bad news,
she explained that the company had been taken over by a big multinational corporation that was
demanding a lot of changes.
So, one day I was in a comfortable job with a generous salary and plenty of annual leave and
the next day I was unemployed. While it was nice to have time on my hands, I knew I would have to
find a job fairly quickly. I soon found out that it wasn't so easy. No one was taking on new staff. Every
morning I searched the Classified Ads, but there was nothing suitable, not even part-time positions.
Finally, a friend asked me if I was interested in helping out in her new cake shop. She needed
somebody to deal with customers and to help her with the cake designs. At first, she could only pay me
by the hour at rather a low hourly rate. To my surprise, I love the work.
I've been a cake designer for three years now. I'm so glad I didn't turn down my friend's job
offer. We have so much work and now I'm no longer an employee. My friend and I are now partners in
the best cake shop in town.
1. What is the writer's main aim in writing the text?
A. To advise people how to change the job
B. To talk about his changes in career
C. To describe his new job
D. To talk about differences between two jobs
2. What does the writer say about his past job?
A. It was a hard job but high salary.
B. It was a part-time job with low salary.
C. It was an interesting job at a big multinational corporation.
D. It was a comfortable job with high salary.
3. What did the writer do after losing his job?
A. He ran his own business.
B. He asked his friend for a part-time job.
C. He tried to find another job.
D. He worked part-time job for Classified Ads.
4. What did the writer think about his current job at first?
36
A. He really loved it. B. The salary was low.
C. He was so interested. D. It was hard.
5. What is the current job of the writer?
A. Graphics designer B. Cake designer
C. Personnel manager D. Employee at the cake shop
Passage 23
The term "folk song" has been current for over a hundred years, but there is still a good deal of
disagreement as to what it actually means. The definition provided by the International Folk Music
Council states that folk music is the music of ordinary people, which is passed on from person to
person by being listened to rather than learned from the printed page. Other factors that help shape a
folk song include: continuity (many performances over a number of years); variation (changes in
words and melodies either through artistic interpretation or failure of memory); and selection (the
acceptance of a song by the community in which it evolves).
When songs have been subjected to these processes, their origin is usually impossible to trace.
For instance, if a farm laborer were to make up a song and sing it to a couple of friends who like it and
memorize it, possibly when the friends come to sing it themselves one of them might forget some of
the words and make up new ones to fill the gap, while the other, perhaps more artistic, might add a few
decorative touches to the tune and improve a couple of lines of text. If this happened a few times there
would be many different versions, the song's original composer would be forgotten, and the song
would become common property. This constant reshaping and recreation is the essence of folk music.
Consequently, modem popular songs and other published music, even though widely sung by people
who are not professional musicians, are not considered folk music. The music and words have been set
by a printed or recorded source, limiting scope for further artistic creation. These songs' origins cannot
be disguised and therefore they belong primarily to the composer and not to a community.
The ideal situation for the creation of folk music is an isolated rural community. In such a
setting, folk songs and dances have a special purpose at every stage in a person's life, from childhood
to death. Epic tales of heroic deeds, seasonal songs relating to calendar events, and occupational songs
are also likely to be sung.
1. What does the passage mainly discuss?
A. Themes commonly found in folk music
B. Elements that define folk music
C. Influences of folk music on popular music
D. The standards of the International Folk Music Council
2. Which of the following statements about the term "folk song" is supported by the passage?
A. It has been used for several centuries.
B. The International Folk Music Council invented it.
C. It is considered to be out-of-date.
D. There is disagreement about its meaning.
3. The word "it" in line 7 (near the end of paragraph 1) refers to ….
A. community B. song C. acceptance D. memory
4. Which of the following is NOT mentioned in the passage as a characteristic of the typical folk song?
A. It is constantly changing over time.
B. it is passed on to other people by being performed.
C. It contains complex musical structures.
D. It appeals to many people.
5. The author mentions that published music is not considered to be folk music because...
A. the original composer can be easily identified
B. the songs attract only the young people in a community
C. the songs are generally performed by professional singers
D. the composers write the music in rural communities
Passage 24
In many ways, the increasingly rapid pace of climate change is a direct result of the growth of
the human population. In the last 100 years, the world population has more than tripled, from just

37
under 2 billion at the beginning of the century to nearly 7 billion today. Obviously, this has meant that
the world has needed to produce three times as much food, energy, and other natural resources.
In addition, the average person uses more energy and natural resources than the average person
one hundred years ago, meaning that the rates of consumption are actually much higher than just the
increase in population would imply. For example, it took the world 125 years to use the first one
trillion barrels of oil. The next trillion barrels will be used in less than 30 years, which is almost 5
times as fast, not three.
All of these activities: food production, energy usage, and the use of natural resources,
contribute to climate change in some way. The greater amounts of oil and other fuels burned to create
energy release chemicals which add to global warming. In order to produce more food, farmers cut
down trees to gain more land for their fields. In addition, we cut down trees to build the houses needed
for a larger population. Those trees are an essential part of controlling global warming. These are just
two examples of the impacts that the growing population has on global warming; others are too
numerous to mention.
1. The word "pace" in the passage is closest in meaning to ______.
A. growth B. speed C. problem D. pollution
2. The word "consumption" in the passage is closest in meaning to______.
A. development B. usage C. population D. increase
3. According to the passage, how does food production contribute to global warming?
A. Producing more food leads to growth in the world population.
B. Food production uses many chemicals which add to global warming.
C. Food production requires that the forests be cleared to create farmland.
D. Food production decreases the ability of the air to release heat.
4. Why does the author mention the rate at which oil is being used paragraph 1?
A. To illustrate that we are using resources faster than the speed at which the population is growing.
B. To suggest that most of the problems of global warming are associated with our rapid use of oil.
C. To suggest that our oil is likely to run out sometime in the next thirty years.
D. To contrast the differences in lifestyle between people giving 100 years ago and people living
today.
5. The word others in the passage refers to…. .
A. problems of global warming in the modern world
B. examples of the environmental consequences of population growth
C. ways in which our usage of oil will affect the world climate
D. the reasons why trees are essential in controlling global warming

Passage 25
In the late 1960's, many people in North America turned their attention to environmental
problems and new steel-and-glass skyscrapers were widely criticized. Ecologists pointed out that a
cluster of tall buildings in a city often overburdens public transportation and parking lot capacities.
Skyscrapers are also lavish consumers and wasters of electric power. In one recent year, the
addition of 17 million square feet of skyscraper office space in New York City raised the peak daily
demand for electricity by 120,000 kilowatts – enough to supply the entire city of Albany, New York,
for a day.
Glass-walled skyscrapers can be especially wasteful. The heat loss (or gain) through a wall of
half-inch plate glass is more than ten times that through a typical masonry wall filled with insulation
board. To lessen the strain on heating and air conditioning equipment builders of skyscrapers have
begun to use double glazed panels of glass, and reflective glasses coated with silver or gold mirror
films that reduce glare as well as heat gain. However, mirror-walled skyscrapers raise the temperature
of the surrounding air and affect neighbouring buildings. Skyscrapers put a severe strain on a city's
sanitation facilities, too. If fully occupied, the two World Trade Center towers in New York City
would alone generate 2.25 million gallons of raw sewage each year - as much as a city the size of
Stamford, Connecticut, which has a population of more than 109,000.

38
Skyscrapers also interfere with television reception, block bird flyways, and obstruct air traffic.
In Boston in the late 1960's, some people even feared that shadows from skyscrapers would kill the
grass on Boston Common.
Still, people continue to build skyscrapers for all the reasons that they have always built them -
personal ambition, civic pride, and the desire of owners to have the largest possible amount of rentable
space.
1. The main purpose of the passage is to _____.
A. compare skyscrapers with other modern structures
B. describe skyscrapers and their effect on the environment
C. advocate the use of masonry in the construction of skyscrapers
D. illustrate some architectural designs of skyscrapers
2. According to the passage, what is one disadvantage of skyscrapers that have mirrored walls?
A. The exterior surrounding air is heated.
B. The windows must be cleaned daily.
C. Construction time is increased.
D. Extra air-conditioning equipment is needed.
3. According to the passage, in the late 1960's some residents of Boston were concerned with which
aspect of skyscrapers?
A. The noise from their construction B. The removal of trees from building sites
C. The harmful effects on the city's grass D. The high cost of rentable office space
4. The author raises issues that would most concern which of the following groups?
A. Electricians B. Environmentalists
C. Aviators D. Teachers
5. Which of the following is NOT mentioned as the reason for building skyscrapers?
A. to modernize B. to fulfill ambition
C. to have more space to rent D. to feel proud

Passage 26
A survey is a study, generally in the form of an interview or a questionnaire that provides
information concerning how people think and act. In the United States, the best-known surveys are the
Gallup poll and the Harris poll. As anyone who watches the news during presidential campaigns
knows, these polls have become an important part of political life in the United States.
North Americans are familiar with the many "person on the street" interviews on local
television news shows. While such interviews can be highly entertaining, they are not necessarily an
accurate indication of public opinion. First, they reflect the opinions of only those people who appear
at a certain location. Thus, such samples can be biased in favor of commuters, middle-class shoppers,
or factory workers, depending on which area the news people select. Second, television interviews
tend to attract outgoing people who are willing to appear on the air, while they frighten away others
who may feel
intimidated by a camera. A survey must be based on a precise, representative sampling if it is to
genuinely reflect a broad range of the population.
In preparing to conduct a survey, sociologists must exercise great care in the wording of
questions. An effective survey question must be simple and clear enough for people to understand it. It
must also be specific enough so that there are no problems in interpreting the results. Even questions
that are less structured must be carefully phrased in order to elicit the type of information desired.
Surveys can be indispensable sources of information, but only if the sampling is done properly and the
questions are worded accurately.
There are two main forms of surveys: the interview and the questionnaire. Each of these forms
of survey research has its advantages. An interviewer can obtain a high response rate because people
find it more difficult to turn down a personal request for an interview than to throw away a written
questionnaire. In addition, an interviewer can go beyond written questions and probe for a subject's
underlying feelings and reasons. However, questionnaires have the advantage of being cheaper and
more consistent.
1. What does the passage mainly discuss?
39
A. The history of surveys in North America
B. The principles of conducting surveys
C. Problems associated with interpreting surveys
D. The importance of polls in American political life
2. The word "they" in paragraph 2 refers to ______.
A. North Americans B. news shows
C. interviews D. opinions
3. According to the passage, the main disadvantage of person-on-the-street interviews is that they
______.
A. are not based on a representative sampling
B. are used only on television
C. are not carefully worded
D. reflect political opinions
4. According to paragraph 3, which of the following is most important for an effective survey?
A. A high number of respondents
B. Carefully worded questions
C. An interviewer's ability to measure respondents' feelings
D. A sociologist who is able to interpret the results
5. According to the passage, one advantage of live interviews over questionnaires is that live
interviews ___.
A. cost less
B. can produce more information
C. are easier to interpret
D. minimize the influence of the researcher

Passage 27
The Nobel Peace Prize is awarded annually and the first woman to win this prize was Baroness
Bertha Felicie Sophie von Suttner in 1905. In fact, her work inspired the creation of the Prize. The first
American woman to win this prize was Jane Addams, in 1931. However, Addams is best known as the
founder of Hull House.
Jane Addams was born in 1860, into a wealthy family. She was one of a small number of
women in her generation to graduate from college. Her commitment to improving the lives of those
around her led her to work for social reform and world peace. In the 1880s Jane Addams traveled to
Europe. While she was in London, she visited a "settlement house" called Toynbee Hall, inspired by
Toynbee Hall, Addams and her friend, Ellen Gates Starr, opened Hull House in a neighborhood of
slums in Chicago in 1899. Hull House provided a day care center for children of working mothers, a
community kitchen, and visiting nurses. Addams and her staff gave classes in English literacy, art, and
other subjects. Hull House also became a meeting place for clubs and labor unions. Most of the people
who worked with Addams in Hull House were well educated, middle-class women. Hull House gave
them an opportunity to use their education and it provided a training ground for careers in social work.
Before World War I, Addams was probably the most beloved woman in America, in a
newspaper poll that asked, "Who among our contemporaries are of the most value to the community?",
Jane Addams was rated second, after Thomas Edison. When she opposed America's involvement in
World War I, however, newspaper editors called her a traitor and a fool, but she never changed her
mind. Jane Addams was a strong champion of several other causes. Until 1920, American women
could not vote. Addams joined in the movement for women's suffrage and was a vice president of the
National American Woman Suffrage Association. She was a founding member of the National
Association for the Advancement of Colored People (NAACP), and was president of the Women's
international League for Peace and Freedom. Her reputation was gradually restored during the last
years of her life. She died of cancer in 1935.
1. With which of the following subjects is the passage mainly concerned?
A. The first award of the Nobel Peace Prize to an American woman.
B. A woman's work for social reform and world peace.
C. The early development of Social Work in America.
40
D. Contributions of educated women to American society.
2. The word "commitment" in line 8 is closest in meaning to….
A. involvement B. obligation C. dedication D. enthusiasm
3. Jane Addams was inspired to open Hull House because ______.
A. it gave educated women an opportunity to use their education and develop careers in social
work.
B. she traveled to Europe in the 1880s.
C. she visited Toynbee Hall.
D. she was invited by a 'settlement house' in Chicago.
4. The word "their" in paragraph 2 refers to…..
A. children of working mothers B. middle-class women
C. visiting nurses D. labor union members
5. According to the passage, Jane Addams' reputation was damaged when she ______.
A. allowed Hull House to become a meeting place for clubs and labor unions
B. joined in the movement for women's suffrage
C. became a founding member of the NAACP
D. opposed America's involvement in World War I

Passage 28
Patricia Denning stars in a sad movie about ambition and heartbreak.
When Cecilia leaves college in Texas, she has everything: a wonderful boyfriend, Carl, an
interesting job and an expensive house in a pretty town. Her life is perfect. Even her black Labrador
dog, Perky, is a star: he wins the local dog competition and Hollywood wants to put him in a movie.
Cecilia goes to Hollywood with him and leaves her boyfriend alone back home. When the
studio fires Perky, Cecilia decides to stay in Hollywood and tries to get other movie roles for her
beautiful dog.
Carl gives Cecilia an ultimatum: come home and stop behaving crazily, or stay there forever.
Cecilia decides to stay in Hollywood and wants to become an actress herself. She sends Perky back to
Texas and starts an acting course. After six months, Cecilia realizes she can't act and she misses home,
but it's too late. Carl has found another girlfriend. Cecilia tries to win him back, and also discovers a
lot about herself on her long journey.
This is a very sad movie, which makes you think about what we want from life. Patricia
Denning is very convincing as Cecilia, but the star of the movie is gorgeous Perky, the black Labrador
who steals our hearts. Black Hollywood is in cinemas from next week.
1. What does Cecilia not have?
A. A house she loves B. A wonderful husband
C. A pet D. A great job
2. When does her wonderful adventure begin?
A. When her dog wins a competition.
B. When she marries her boyfriend, Carl.
C. When she goes on vacation to Hollywood.
D. When she buys the house in a pretty town.
3. What happens to Perky in Hollywood?
A. He isn't very successful. B. He becomes a star.
C. He's too nervous to work. D. He makes two movies and retires.
4. What is Carl's reaction to Cecilia living in Hollywood?
A. He doesn't know she went. B. He's angry.
C. He doesn't care. D. He's supportive.
5. Why does Cecilia stay in Hollywood?
A. She wants to become an actress. B. She is too busy making movies.
C. She hates Carl. D. She has no money.

41
PART 4 – LISTENING

PART I - Listen to the recording and choose the best answer to each question.

Recording 1
1. Who is Owen Winters?
A. The client of DreamsRreal.com. B. The foundder of DreamsRreal.com
C. The CEO of DreamsRreal.com D. The MC of the programme
2. How does the DreamsRreal work?
A. To help people find jobs B. To help people mangae their lives
C. To help people make their dreams come true D. To help people build up dreams
3. What did the first client want?
A. to appear with a rock star in a live concert. B. people to recognise her talent.
C. to play in front of a large number of people. D. to get the first prize in a music contest.
4. Why does Owen Winter find his job at DreamReal.com easy?
A. It’s similar to his previous job.
B. He has worked in business for many years.
C. He knows a lot of people in the music business.
D. He likes working with singers.
5. How many fans in the concert did DreamsRreal have to pay?
A. All of them B. None of them C. Most of them D. Some of them

Recording 2
1. What is the purpose of the call?
A. To make a reservation B. To enquire about a reservation
C. To get the reference booking D. To arrange an appointment
2. When does the man want to change the dates to?
A. A week later B. Two weeks later C. One day later D. Two days later
3. Who does the woman need to ask?
A. Her boss B. Her manager C. Her supervisor D. Her colleague
4. What is the hotel’s policy?
A. Online special deals are refundable but non-tranferable
B. Online special deals are non-refundable bit transferable
C. Online special deals are refundable and transferable
D. Online special deals are non-refundable and non-transferable
5. What does the man want to do next?
A. Cancel the booking B. Speak to the woman’s supervisor
C. Get a better price D. Meet the hotel’s manager

Recording 3
1. What is the aim of the project?
A. To prevent malaria B. To prevent diabetes
C. To prevent pneumonia D. To prevent heart attack
2. What is the situtation of the disease?
A. Increasing rapidly B. Levelling off
C. Deacreasing rapidly D. Decreasing slowly
3. What were the temperatures at the desert?
A. Fourteen degrees Celsius B. Forty-five degrees Celsius
C. Forty degrees Celsius D. Forty-nine degrees Celsius
4. Why was the tour operator arrested?
A. For charging too much B. For not having a business permit
C. For conducting a too big group D. For not helping rourists
5. What has the USA resently done?

42
A. Cut taxes on all goods from abroad B. Decreased taxes on all goods form abroad
C. Increased taxes on all goods from abroad D. Put taxes on all goods from abroad

Recording 4
1. Why does the woman support the idea of allowing students to use their phones in class?
A. Because phones are important to them
B. Because they would be more motivated
C. Because they would be more concentrated
D. Because they would be able to see short movies
2. What does the man think of the students?
A. They make phone calls too much
B. They get bored easily
C. They phone each other when they are bored
D. They text each other when they are bored
3. The woman thinks phones could be useful for...........
A. practising languages B. doing homework
C. keeping in touch D. teaching languages
4. What is the man worried about using phones in class?
A. Laziness in class B. Bullying and thieves
C. Breaking school rules D. Cheating in exams
5. At last, the woman........
A. changed her mind B. felt worried C. kept her mind D. felt confused

Recording 5
1. What did the client want to do?
A. Fly across Pacific Ocean on a supersonic aeroplane
B. Fly across Atlantic Ocean on a supersonic aeroplane
C. Fly on a spacecraft
D. Fly an aeroplane
2. What kind of plane did they use?
A. A Concorde B. A Boeing C. A helicopter D. An air force plane
3. How did the person with Tom Cruise make-up feel about the experience?
A. He loved all the attention. B. He enjoyed it.
C. He didn’t enjoy it. D. He didn’t like the bodyguards.
4. What happened to the woman who wanted to go into the space?
A. Her request was not possible at first, but now she’s going to achieve her dream.
B. She couldn’t afford it at first but now she can.
C. It has taken DreamsRreal.com a long time to plan and organize the trip.
D. She had to find someone to go with her.
5. What can be inferred from the interview about his company?
A. Sometimes, they say no to a dream. B. They try their best to deal with all requests.
C. They charge very much. D. They don’t work with ordinary people.

Recording 6
1. What is the purpose of the call?
A. To book a course B. To enquire about a course
C. To cancel the booking D. To make an appointment
2. How did she book the course?
A. Online B. From the office
C. From a marketing staff D. From an intermediate friend
3. What course did she book?
A. Intermidiate general English B. Pre-intermediate general English
C. Advanced general English D. Pre-advanced general English

43
4. Which group does she want to change to?
A. Advanced business English B. Advanced general English
C. Pre-advanced business English D. Intermidaite business English
5. What does she have to do to change the course?
A. Pay an extra fee B. Do a test online
C. Do a test in person D. Do a test on the phone

Recording 7
1. Why is the man against cosmetic surgery?
A. Because it is expensive. B. Because it is dangerous.
C. Because it takes a lot of time. D. Because it doesn’t last for long.
2. What did Mike’s girlfriend do in her surgery?
A. She had some Botox injections. B. She had a filter nose.
C. She had some false teeth. D. She had her legs stretched.
3. What happened to Mike’s girlfriend?
A. She couldn’t eat anything. B. She couldn’t smell anything.
C. She couldn’t talk. D. She couldn’t smile.
4. The man thinks that people may have cosmetic surgery for ________.
A. appearance reasons B. health reasons C. age reasons D. career reasons
5. The woman thinks ________.
A. she will never get cosmetic surgery
B. the man should have cosmetic surgery
C. she might have cosmetic surgery if she gets health problems
D. the man should stop criticizing cosmetic surgery

Recording 8
1. The man doesn’t support the idea of ________.
A. downloading music illegally B. listening to music on the Internet
C. paying for the track or album online D. uploading music on the Internet.
2. The woman thinks musicians ________.
A. get a lot of money from CD sales B. become well-known from CD sales
C. get little money from CD sales D. want people to pay for CDs
3. The woman thinks famous musician do not need ________.
A. money B. fame C. managers D. companies
4. The woman thinks singers and groups make money from ________.
A. CD sales B. endorsement C. investment D. concerts
5. After talking with the woman, the man ________.
A. agrees with her B. wants her to be fined
C. still keeps his point of view D. hates her

Recording 9
1. Who needs to be in each team?
A. Two musicians B. Two teachers C. Two celebrities D. Two companies
2. Besides the swimming pool, what is there in the studio?
A. A giant wall B. A giant ball C. A giant door D. A giant bed
3. What does the host say?
A. Drink the water B. Bring on the wall
C. Throwing the ball D. Shrinking to the door
4. What do the players have to do?
A. Catch the ball B. Hit the wall
C. Get through the hole D. Jump into the swimming pool.
5. The woman is possibly describing a..........
A. film B. swimming pool C. lesson D. game

44
Recording 10
1. How many people are there in the game?
A. five B. nine C. ten D. twelve
2. Where does each player stand?
A. In front of the door B. In the middle of the studio
C. Beside a play station D. Behind a metal desk
3. What do they get if they answer the questions correctly?
A. money B. gifts C. medals D. applause
4. What happens if someone gives a wrong answer to the question?
A. The team has to stop the game. B. The team loses half of the money.
C. The team loses all of the money. D. The team has to pay the host.
5. What happens after each round?
A. One person gets eliminated. B. All the players go to the next round.
C. All the team’s money is set at zero. D. The host is change.

Recording 11
1. Who is probably giving this talk?
A. An artist. B. A tour guide. C. An Indian. D. Orville Wright.
2. How many items are on exhibit in the Smithsonian Museums?
A. Several. B. Sixty thousand.
C. Sixteen million. D. Millions and millions.
3. According to the talk, which museum has exhibits of early Eskimos?
A. The National Air and Space Museum.
B. The Museum of Natural History.
C. The American History Museum.
D. The Smithsonian Arts and Industries Building.
4. Which museum will they visit as a group?
A. The American History Museum.
B. The Smithsonian Arts and Industries Building.
C. The Washington Museum.
D. The National Air and Space Museum.
5. Where will they go tomorrow?
A. To the White House. B. To the Smithsonian.
To the mall. To various other museums.

Recording 12
1. How does the woman describe her Wyoming vacation?
A. Boring. B. Fantastic. C. Lengthy. D. Faithful.
2. How did the woman probably travel on vacation?
A. By car. B. By plane. C. By train. D. By bicycle.
3. Which of the following did the woman do on vacation?
A. She went directly to Yellowstone.
B. She spent a few weeks in Laramie.
C. She stopped at the Devil’s Tower National Monument.
D. She made a few stops before going on to Yellowstone.
4. Which of the following is a well-known sight in Yellowstone?
A. Laramie. B. Devil’s Tower National Monument.
C. Old Faithful. D. Wyoming.
5. What would the man like to do?
A. Hear again about Yellowstone. B. Take a trip to Yellowstone.
C. Get a job in a national park. D. Move to Yellowstone.

Recording 13
1. What does the man want to do?
45
A. Find work on campus B. Work in the employment office
C. Help students find jobs D. Ask the woman questions
2. Where does the conversation probably take place?
A. In the library. B. In a classroom. C. In a campus office. D. In an apartment.
3. How many hours of work does the man want per week?
A. No more than ten. B. At least twenty. C. Not more than twenty. D. Up to ten.
4. When can the man work?
A. Every morning. B. Afternoon and weekends.
C. When he’s in class. D. Weekdays.
5. What does the woman tell the man to do tomorrow?
A. Fill out a form. B. Give her some additional information.
C. Tell her some news. D. Phone her.

Recording 14
1. What is the main idea of the talk?
A. How to be a good actor. B. What is a good performance?
C. Causes of stage fright. D. Stage fright and how to avoid it.
2. Who would probably NOT experience stage fright in their work?
A. Writers. B. Actors. C. Athletes. D. Musicians.
3. What physical reaction might someone who is experiencing stage fright commonly have?
A. He or she would see butterflies. B. He or she would break a leg.
C. He or she would have shaky knees. D. He or she would stop breathing.
4. How can someone combat the negative thoughts associated with stage fright?
A. By staring at the audience.
B. By breathing shallowly.
C. By thinking about possible negative outcomes.
D. By focusing on what needs to be done.
5. When should the actors arrive at the theatre?
A. At two o’clock. B. At four o’clock. C. At six o’clock. D. At eight o’clock.

Recording 15
1. What is the topic of the talk?
A. Soft, warm clothing. B. Problems in landfills.
C. How fleece is obtained. D. Recycling soda bottles.
2. In the past, what commonly happened to the soda bottles?
A. They were left in landfill areas. B. They were reused.
C. They were recycled. D. They were refilled.
3. Where are the soda bottles gathered now?
A. In a plant. B. In a warehouse.
C. In a manufacturing company. D. By the sea.
4. What can be made out of the soda bottles?
A. Dye. B. Warm, soft clothing. C. Computer chips. D. Glass bottles.
5. What does the speaker recommend?
A. Buying plastic bottles. B. Solving the problems in landfills.
C. Buying these recycled products. D. Becoming aware of the environment.

Recording 16
1. What information surprised the man?
A. That babies sleep 13 hours a day.
B. That the woman was taking a psychology class.
C. That more mature people required so much sleep.
D. That the need for sleep decreases with age.
2. Where did the woman learn about this information about sleep?

46
A. In psychology class. B. In a discussion with the man.
C. From the article that she read. D. From the class textbook.
3. About how many hours do one-year-old children sleep a day?
A. 13 B. 20 C. 12 D. 10
4. What is the man’s age group?
A. Teens. B. Twenties. C. Thirties. D. Fifties.
5. Approximately how many hours of sleep per night do 50-year-olds require?
A. 13 hours. B. 9 hours. C. 8 hours. D. 6 hours.

Recording 17
1. What is the woman planning to do on Saturday?
A. Attend a football game alone. B. Go to a sporting event.
C. Eat in the cafeteria and study. D. See a play.
2. Why does the man want to go to the football game?
A. It’s the final game of the season. B. It's better than the drama department’s play.
C. It’s a very important game. D. It’s close to the cafeteria
3. When will they be meeting for dinner?
A. At 6 on Saturday night. B. At 6 on Sunday night.
C. In the late afternoon. D. When the game is over.
4. What is at the same time as the music department’s concert?
A. A play. B. A game.
C. A study group meeting. D. Dinner in the cafeteria.
5. When does the man plan to study?
A. Saturday night. B. After dinner in the cafeteria.
C. Sunday afternoon. D. Maybe next weekend.

Recording 18
1. Who is Ms. Martin talking to??
A. Other librarians.
B. Undergraduate students.
C. Students who are not in the business department.
D. Graduate business students.
2. What are the library’s hours during final exam week?
A. It opens at 7:00 a.m. B. It closes at 7:00 p.m.
C. It closes at midnight. D. It is always open.
3. Where are the computerized search facilities?
A. On the first floor B. On the second floor
C. On the seventh floor D. On the floor above the computer area
4. What two areas will the tour concentrate on?
A. Computer area and business materials. B. Magazines and newspapers.
C. Business department and library staff office. D. First and second floors of the library.
5. What are the students probably going to do next?
A. Go home. B. Return to class.
C. Work on the computers. D. Tour the library.

Recording 19
1. What does the man ask the woman to do?
A. To write his paper. B. To help him decide on the topic.
C. To teach him about history. D. To discuss history with him.
2. What is History 101 about?
A. Revolution War B. Civil War C. World War I D. American history
3. When in the semester does this conversation probably take place?
A. At the beginning of the semester. B. Before the start of the semester.
47
C. Near the end of the semester. D. One week after the semester is finished.
4. Why won’t the man choose ‘technology’ as a topic?
A. The topic is too general.
B. He isn’t interested in technology.
C. He doesn’t have enough time.
D. Technology has nothing to do with American history.
5. How much time does the man have to write the paper?
A. A month. B. The semester. C. 7 days. D. A days or two.

Part II: Listen to the recording and fill in the gaps with the words you heard. Write NO MORE
THAN THREE words for each gap.

Recording 1
I decided to do this because I hate it when people forget my name, like at school the teachers who
don’t know your name, they don’t give you so much (1)....................................... So anyway, I looked
on some websites to find out the best way to do it. Apparently, there are two
(2)............................................things, first is that when you’re introduced, you really pay attention and
look at the person and try to find a way to (3)........................................the namess. For example, I
recetly met a woman called Keira and she had curly hair, so Keira, curly, sounds similar, you see what
I mean. That was easy. Then secondly, you need to repeat the name as often as possible, say it to
yourself (4).......................................times and use it when you’re talking to the person. You just have
to be careful that you don’t sound really strange. Anyway, the result’s been good. Somehow, peole
seem friendlier and I fell a lot more (5)............................................. about chatting to people. The only
problem is someone told me it made people uncomfortable because they couldn’t remember my name.

Recording 2
It was quite difficult at first...I mean you actually have to stop people trying to give you one. I didn’t
realise before I started how many are given out all the time. I thought this was a good thing to do
because apparently it can take up to a (1).............................................. for one to decay and about
thirteen billion are given out each year in the UK alone. And it’s not only the pollution but
(2)........................................ and fish can get caught in them. Anyway, I invested in two shopping bags
and I’ve been using them for the past three (3).......................... The only problem is I keep forgetting to
take them out of the house or leave them in the car, which is very annoying. My (4).............................
has been to get one of those fold-up bags that you can carry in your pocket or bag. I’ve got all my
friends to do the same and now our local(10).............................. are going to become a plastic-bag-free
zone. At least that will make me remember to take a bag!

Recording3
I decided to combine two of the ideas. I’ve been hopeless at telling jokes. I’m sure it’s not because I
don’t have a snese of (1)......................................... It’s something about the timing. And I know that
jokes are great for building(2).............................................. and good for me personally as I often have
to give business presentations and a funny story really helps build (3)............................................ with
the audience. One of the best things about doing this was I asked all my friends to tell me their
(4)..........................................joke and we had lot of laugh-out-loud times together. And I’m getting
better, though I did have one very (5)..................................... moment at work when I told my joke to
my boss and he stared at me like I was an idiot. You want to hear a joke? Something short? Ok....What
do cows do on Saturday nights? They rent mooooooovies.

Recording 4
The way it works is that you (1)...................................a grid of 5 x 5 on a piece of paper. There are two
players and the object of the game is to complete the (2)..................................... “SOS” in a straight
line as many times as you can. So, the first thing you do is one of you writes an “S” or an “O” in one
of the squares. Then the other player writes an “S” or an “O” in another square. (3)..............................
one of you completes an “SOS”, you get another turn and basically, the point is not to let your partner
48
succeed, so what usually happens is that one player gets an “SOS” and then (4)........................... the
other player. It’s easy to lose track of who is winning, so the key thing is to keep score of who gets
how many “SOSs”. Then after you’ve (5)...................... (once the grid is full), the winner is the player
with the most “SOSs”.

Recording 5
I thought this was a good one to try because everyone always looks too bored or
(1)......................................., especially on public transport. So, the next time I was sitting on a train
and someone sat opposite me I looked up and gave them a big (2)...................................... They looked
a bit surprised but smiled back at me, then buried their face in the (3)............................ I got the
impression they were a bit embarrassed. Anyway, I continued and kept smiling at all sorts of people
during the day. To be honest, I got a mixed (4)..........................................., but the kids and older
people seemed the friendliest. Oh, and I found out later that one woman in the office thought I was
(5).................................... with her!

Recording 6
I’m logged on twenty-four hours a day, and each time a message comes in I check it...I start getting
(1)................................. if I don’t get at least one an hour. Nowadays, I often don’t
(2)............................... the phone when my old friends call. Almost all my friends are people I’ve met
(3)........................................... - the other day I met some people I knew from uni and I actually found
it quite (4)............................................. talking to them face to face because I’m much more used to
(5).......................................... with people online. I’m a bit worried because my eyes are starting to
hurt...

Recording 7
I'm going to tell you about Robo-Chef. Basically, Robo-Chef can (1)........................................ and
cook all your favourite recipes. It works like this. First of all, it washes and prepares all the
(2)........................................, then it prepares your dish and cooks it for you on your cooker. Robo-Chef
comes complete with hundreds of menus already (3)......................................... But you can also
programme Robo-Chef with your own recipes, or if you want to try something new, you can download
new recipes whenever you like. All you have to do is (4)........................................ the dish you want,
decide how many people you want Robo-Chef to cook for and what time you want the meal to be
ready. So, let's say you would like a vegetable lasagna for six people, ready by eight o'clock. Then, just
make sure you have all the (5)........................................ in the kitchen, press the button and that's it.

Recording 8
One of the most, er, memorable moments, or not moments rather events, in my life ... er ... was a
(1)........................................ of years ago. Erm ... It all started one day when I was at work and my
brother phoned me out of the blue and said, um, 'What are you doing the weekend of Sept 23rd?' or
(2)........................................ it was. And I said, 'I don't know.' He said, 'Well, book a flight to Norway.'
My brother lives in Norway and I live in England so I said, 'Why?', he said, 'Oh I'll let you know when
you get there - it's a (3).........................................' So, weeks went on and I tried to work out what this
could be, but I had absolutely no idea. So the weekend in question came about ... went to
(4)........................................., got on my flight and ah I was met there by somebody I'd never met
before. He just came up and said, 'Are you Stig?'. I said, 'Yes.' And he said, 'OK. Come with me.' So, I
went with him to the car. We drove for a little while and I tried to kind of get it out of him where we
were going, but he wouldn't tell me anything. He pulled up (5)........................................ a hotel and
there were my brother and my half-brother and my two half-sisters there waiting for me.

Recording 9
It's interesting: one of the most (1).................................. people I know is a ten-year-old boy from
Egypt. He doesn't go to school and he works on a street in Cairo. in one of the
(2)..................................... areas. And he sells things like small statues of the pyramids, things like that,
to tourists. Now, the reason I say he's intelligent is that he can sell you something in about fifteen
49
(3)........................................ I once spent an afternoon watching him and it was
(4).................................... Most of the time he uses English, but he guesses where you're from by
looking at you and then he starts speaking. He can speak just a little bit of French, Spanish,
(5)........................................, Italian, German, etc. It's amazing. He knows just enough in all these
languages to say hello and sell you something.

Recording 10
A couple of years ago, er, I learned how to scuba dive which was, um, really exciting, really good
(1)........................................ and when you're learning half of the, the training is in the classroom and
half is, er, a (2)........................................ in a swimming pool. So, the classroom stuff was fine, erm. I
found it really quite easy. I was learning with my mum and she was really worried about doing the
kind of more (3) ........................................ stuff and passing the exam but I found that part OK. It was
the practical stuff that I had trouble with and she was really lucky, she was erm, really good. But you
go and you learn all the (4).................................... stuff, you know how to go under the water, how to
clear your mask if you get water in it, that kind of thing. And then you have to do two
(5).................................... outside in a, in a kind of reservoir or a quarry or, you know, something like
that.

Recording 11
My best friend Katy lives near a beautiful white sand beach. Her parents take us there every
summer. The water is (1) ……………………. We have a lot of fun in the waves. Last year, there was
an oil spill at Katy’s beach. We volunteered at the beach to clean up the oil and help the birds that were
caught in the spill.
When we arrived at the beach that day, the water was slick with oil and the white sand had
turned black. It was an ugly sight. At first, the (2) …………………… would not let us on the beach.
We had to watch from a distance. We watched as they put a huge float in the water, called a boom.
This device kept the oil slick (3) ……………………. Once they stopped the slick, we were allowed on
the beach to help. Each of us was given a broom. We swept the oil into a pile. The workers took it
away in big trucks. We all worked very hard that day to (4) ……………………. Once the oil was
removed, the birds that had been caught in the oil had to be helped. The workers showed us how to
remove the oil from their wings. Then the birds could fly away to (5) ……………………. We were
very proud of ourselves that day. We helped save our beautiful beach and the birds who visit it.

Recording 12
The Amazon is an amazing river. It is unique in many ways. It is the second longest river in the
world. It is (1) ……………………. . It starts up in the Andes Mountains. Its drainage basin (the area
of land that collects water) covers 40 percent of South America. The Amazon River has many small
rivers that (2) …………………… it with water. It is the widest river in the world. The river rises more
than nine meters during the (3) ……………………. It overflows its banks. It floods large areas of the
rainforest. During that time, in some parts, it has a width of up to 40 kilometers. The Amazon River is
over 325 kilometers wide at its mouth. It has more water than any other river in the world. The
Amazon River is also home to (4) ……………………. Over 2,000 types of fish live in the Amazon
River. One kind of fish is the piranha. The piranha is a terrifying fish. A piranha grows up to 60
centimeters long. It hunts in a group. A group of piranhas can kill and eat cattle in a matter of minutes.
They are so (5) …………………… that only the bones are left. This fish is one of the most dangerous
creatures in the Amazon River.

Recording 13
There are more than six billion human beings in the world. Each one is different from all the
others. No two people look (1) ……………………. People from one country may look the same in
general. But they are not totally alike. Most people in Vietnam have (2) …………………… and black
eyes. But they all look different. People in America have straight hair, curly hair, or wavy hair. Their
hair may be black, brown, yellow, or red. Some people have brown or gray eyes. Some have blue or
green eyes. Even people in the same family (3) …………………… from each other!
50
We all think differently. We all like different things. We like different kinds of food and
different kinds of music. We also like (4) …………………… of clothing. Wherever you are, you are
different from everyone else. Nobody else looks just like you. Nobody has your (5)
…………………… or your talents. There are more than six billion people on Earth, but none of them
are exactly like you. You are one of a kind!

Recording 14
There are many kinds of schools for many kinds of students. One kind of school is the home
school. Home schools are exactly that schools at home. Parents choose to home school their children
for (1) ……………………. Some people feel that public schools are too dangerous. Some people
think that the (2) …………………… is too low. Some parents do not approve of the (3)
…………………… their children learn in public schools. If parents choose to home school their
children, they must be ready to teach them well. There are many benefits to home schooling. Home
schools are very small. There are just one or two students. The student’s mother or father is the
teacher. Another benefit is that students can study interesting things. Some people think that home
schooling is not good for students. They believe public schools teach children (4) …………………….
They feel that public schools give a (5) ……………………. In the end, parents must decide if home
schooling is the right choice for their children.

Recording 15
Welcome on board the (1) ……………………. We are now leaving Paris. We will arrive in
Marseilles at 3:00 p.m. We offer (2) …………………… in each car. Please have as much coffee, tea,
or soft drinks as you wish. There is a restaurant on the train. You may purchase a meal there during the
trip. The restaurant is in car 11. We will make two stops on this trip. We will pick up passengers in
Provence and Dijon. Passengers (3) …………………… can ask the attendant for a free timetable. This
will help you with your onward trip. Please note that smoking is only (4) …………………… in cars
15 and 23. Newspapers are given to all passengers. Mobile phones and the Internet may be used in cars
(5) ……………………. If you need assistance, please press the button near your seat to contact a
cabin attendant. Please sit back, relax, and enjoy the trip. We are pleased to have you on board. We
look forward to serving you again in the future.

Recording 16
Okay... Now let's talk about test and grades. There will be two (1) …………………… and one
oral test this semester. Each written test will have 40 multiple choice questions. The oral test will have
(2) ……………………. Uh... Some of you may think the tests are too difficult with too many
questions. But that is not so. As a matter of fact, there will be a (3) …………………… every class this
semester. And if you study them well, then the tests will be very easy. And speaking about grades, the
written and oral tests will be just a small portion of your (4) ……………………. Active class
participation is the most important thing to get a good grade. So, please listen to my lectures carefully,
ask many questions, and (5) …………………… with your classmates and me during the class. It will
be very good for you to prepare for each unit before class too.

Recording 17
Hello, my name is Jenny Smith. I will be your physical geography teacher for this (1)
……………………. Um... Since this is your first class, I want to talk about what physical geography
is. What is physical geography? Does anybody have any ideas? Basically, it's a science about the earth.
But it differs from others in that it (3) …………………… from other sciences profoundly. For
example, you will find information from (3) …………………, …………………., climatology,
astronomy and many other sciences in physical geography. Does it sound scary? Actually, you don't
really need to worry about it. Fundamentally, this is a (4) …………………… . So, we just will use
what we already know in these areas. We will not go into the details of those subjects. Okay... now let
me show you some examples... turn to page 11. Chapter one. (5) …………………….

Recording 18
51
Okay, class. These are the four (1) …………………… you have to remember during this
course. Now I'm going to tell you once again. So please remember, and do not forget. Okay? First, you
have to be prepared to (2) …………………… of the readings before each class. Second, during the
class, you have to ask and answer each other about the contents. Sometimes you have to (3)
…………………… a person when you disagree with him or her. Third, every two weeks, you need to
choose one of the subjects on the syllabus and prepare an (4) ……………………. And finally, on the
last day of the term, you have to submit a course paper. Papers must be more (5) ……………………
than the broad topics discussed in class. Hmm... Now, I know how all you guys must feel now. It
sounds really difficult, doesn't it? But these are the rules in my class. If you want to get a good grade,
you have to try to meet these requirements.

Recording 19
Okay, class, last time, I told you about the climate. Some of you still may be (1)
…………………… about climate and weather. So, I want to explain the difference again. From the (2)
……………………, weather is an event that occurs for a short period of time. For example, if we want
to take a trip, we are curious about the weather because we go on a trip for just a short period of time.
So, this is weather. Now for climate... Climate is a (3) …………………… of weather, so it affects
people's lives longer compared to weather. For example, sometimes we see people moving to another
country. When they move, they want to know what the (4) …………………… is like because they
will live there for a certain period of time. So, this is climate. There are many types of climates on
Earth. The climate (5) …………………… over a long period of time like years, decades, or even
centuries. Now I hope you aren't confused about the climate anymore.

Recording 20
Okay, now let's talk about handball violations. I bet you all know that you aren't supposed to
(1) …………………… when you play soccer. It is called a 'handball violation'. You cannot touch any
part of the ball with your hands unless you are the (2) ……………………. Only the goalkeeper can
touch the ball with his hands. A handball violation includes using any part of the body from the (3)
…………………… to the shoulders. Until here, this is all you know. Now, I want to ask you some
questions. First, picture this situation. In the middle of the game, a player (4) ……………………, and
the ball touches the hand of the opposite (5) ……………………. But he didn't touch it intentionally.
Now, is it a handball violation? I want you to think about it. Here goes another question. What if the
kicker did that intentionally? Is it a handball violation?

52
NGÂN HÀNG ĐỀ THI TỰ LUẬN
PART 1 - WRITING

Test 1: Write a descriptive paragraph of about 150 words on the following task:
Describe a person that impresses you most.
Test 2: Write a descriptive paragraph of about 150 words on the following task:
Describe place that you feel the most comfortable to stay in.
Test 3: Write a classification paragraph of about 150 words on the following question:
What should the government do to improve the quality of population?
Test 4: Write an opinion paragraph of about 150 words on the following question:
Smoking should be banned in all restaurants and bars. Do you agree or disagree?
Test 5: Write an opinion paragraph of about 150 words on the following question:
University students should not have part-time jobs. Do you agree or disagree?
Test 6: Write an opinion paragraph of about 150 words on the following question:
At university, self-studying at home is more important than attending class. Do you agree or
disagree?
Test 7: Write a comparison/contrast paragraph of about 150 words on the following question:
What are the advantages or disadvantages of studying overseas?
Test 8: Write a comparison and contrast paragraph of about 150 words on the following task:
Compare the benefits of living in a small town and living in a big city.
Test 9: Write a cause/effect paragraph of about 150 words on the following question:
What are the effects of obesity?
Test 10: Write a cause/effect paragraph of about 150 words on the following question:
What are main causes of high social evil rate?
Test 11: Write a descriptive paragraph of about 150 words on the following task:
Describe the daily life of a police officer. Is being a police officer a good job? Is it easy?
What kinds of things does a police officer do everyday?
Test 12: Write an opinion paragraph of about 150 words on the following question:
In your opinion, is it easy or difficult to learn English? Why do you think so? Give two or
three strong reasons to support your opinion.
Test 13: Write a definition paragraph of about 150 words on the following question:
We sometimes hear or read that a certain person showed great courage in doing something.
What does the word courage mean to you? Give examples to help your readers better understand
your definition of courage.

53
Test 14: Write a descriptive paragraph of about 150 words on the following task:
Describe your favourite place to study.
Test 15: Write a paragraph of about 150 words on the following question:
What are the necessary qualities for students to study online effectively?
Test 16: Write a paragraph of about 150 words on the following question:
What are the advantages or disadvantages of using public transportation to commute?
Test 17: Write a descriptive paragraph of about 150 words on the following task:
Describe the best place to visit in your hometown.
Test 18: Write a paragraph of about 150 words on the following task:
What are the advantages and disadvantages of working in the city?
Test 19: Write a paragraph of about 150 words on the following task:
What are the benefits of reading books?
Test 20: Write a descriptive paragraph of about 150 words on the following task:
What is your opinion about vegetarian diets? Say whether or not you think a vegetarian diet
is healthy and support your opinion.
Test 21: Write a descriptive paragraph of about 150 words on the following task:
Describe the public transportation services available in your area. You should mention what
is available, what is not available, and give recommendations for improving your local public
transport facilities.
Test 22: Write an opinion paragraph of about 150 words on the following statement:
“If you want to be famous, then you have to accept that no part of your life is ever going to
be private.”
Test 23: Write an opinion paragraph of about 150 words on the following task:
How does growing up in a large family compare with growing up in a small family? What
are the advantages and disadvantages?
Test 24: Write an opinion paragraph of about 150 words on the following task:
“Which is better, living in a city or living in the country?”
Test 25: Write an opinion paragraph of about 150 words on the following statement:
“You can do almost anything with a mobile phone these days.”
Test 26: Write a paragraph of about 150 words on the following task:
What are the advantages and disadvantages of the globalization of education?
Test 27: Write a paragraph of about 150 words on the following task:
Some people think that liking your job is more important than how much money you earn
while others think that your salary is more important than how much you like your job. Compare
the two views and give your opinion.

54
Test 28: Write a paragraph of about 150 words on the following task:
It is sometimes said that old friendships are as valuable as family relationships. Do you
agree or disagree?
Test 29: Write a descriptive paragraph of about 150 words on the following task:
Describe an object that has been in your family for a long time.
Test 30: Write a comparison paragraph of about 150 words on the following task:
Compare living at home and living on campus
Test 31: Write an opinion paragraph of about 150 words on the following statement:
Should public transportation be free for city residents? Some believe that this would help
decrease our carbon footprint. What is your opinion?

55
PART 6 – SPEAKING

HANOI UNIVERSITY OF BUSINESS AND TECHNOLOGY


FACULTY OF ENGLISH LANGUAGE STUDIES
*****
3.1 SPEAKING TEST

Topic 1

Student –Examiner (suggested questions)


Good morning/ afternoon.
Please take a seat. My name is … and I’m your examiner. Can you tell me your full
name, please?
What should I call you?

Part 1: Now, I’d like to ask you some questions about your personality and preference.
1. Do you prefer saying hello first or waiting for people to break the ice when you
meet someone for the first time?
2. What do you like to do when you are at home?
3. Do you prefer staying alone or staying with someone else when you don`t work?

Part 2: I’m going to give you a topic and I’d like you to talk about it for 2-3 minutes.
Before you talk you have one minute to think about what you’re going to say. You can
make notes if you wish.

Describe a flat-mate, a roommate or a classmate that impresses you


most. You should say:
- Who this person is
- How long you have known each other
- What he/she looks like
- What he/she is like
and explain how this person has impressed you.

Part 3: We’ve been talking about a roommate/classmate. Now I’d like to ask you some
more questions about this topic.
1. What problems often occur when two people share a room with each other? What
are the solutions?
2. Why is it difficult for some people to make friends in a new working or learning
environment?
3. What skills do you need to maintain a good relationship with your
roommate/classmate?
It’s been very interesting talking to you. Thank you and good luck.

56
HANOI UNIVERSITY OF BUSINESS AND TECHNOLOGY
FACULTY OF ENGLISH LANGUAGE STUDIES
*****

3.1 SPEAKING TEST

Topic 2

Student –Examiner (suggested questions)


Good morning/ afternoon.
Please take a seat. My name is … and I’m your examiner. Can you tell me your full
name, please?
What should I call you?

Part 1: Now, I’d like to ask you some questions about your personality and preference
1. Do you prefer working alone or working in group? Why?
2. What are the advantages/disadvantages of working with a partner/a group?
3. Do you like solving problems? Why (not)?

Part 2
I’m going to give you a topic and I’d like you to talk about it for 2-3 minutes. Before
you talk you have one minute to think about what you’re going to say. You can make
notes if you wish.

Describe your first day at work, at school or at university that makes


you unforgettable. You should say:
- When/where it was
- What happened on that day
- How you felt at that time

and explain why you never forget that day.

Part 3
We’ve been talking about your first day at school/at work. Now I’d like to ask you some
more questions about this topic.
1. Is it a good idea to start the conversation first when you are a newcomer? Why
(not)?
2. What should you prepare for the first day at work?
3. If your classmates/colleagues did not have good first impression on you, what
would you do to change their mind?

It’s been very interesting talking to you. Thank you and good luck.

57
HANOI UNIVERSITY OF BUSINESS AND TECHNOLOGY
FACULTY OF ENGLISH LANGUAGE STUDIES
*****

3.1 SPEAKING TEST

Topic 3

Student – Examiner (suggested questions)


Good morning/ afternoon.
Please take a seat. My name is … and I’m your examiner. Can you tell me your full
name, please?
What should I call you?

Part 1: Now, I’d like to ask you some questions about your emotion.
1. How you ever been in very bad mood? When and why?
2. What do you often do when you are in bad mood?
3. What kind of situations often affects your emotion and mood the most?

Part 2
I’m going to give you a topic and I’d like you to talk about it for 2-3 minutes. Before
you talk you have one minute to think about what you’re going to say. You can make
notes if you wish.

Describe the best thing that you have just bought. You should say:
- Where and when you bought it
- What it looks like
- What you have used it for

and explain why it is the best thing you have ever had.

Part 3
We’ve been talking about your best thing. Now I’d like to ask you some more questions
about this topic.
1. What decides the value of something: its uses or its price?
2. If you bought something online and it turned out to be a fake one, what would
you do?
3. When you buy something, what factors do you often consider?

It’s been very interesting talking to you. Thank you and good luck.

58
HANOI UNIVERSITY OF BUSINESS AND TECHNOLOGY
FACULTY OF ENGLISH LANGUAGE STUDIES
*****

3.1 SPEAKING TEST

Topic 4

Student –Examiner (suggested questions)


Good morning/ afternoon.
Please take a seat. My name is … and I’m your examiner. Can you tell me your full
name, please?
What should I call you?

Part 1: Now, I’d like to ask you some questions about your emotion and life
experiences.
1. How do you figure out what people mean when they use gestures and body
language?
2. Have you ever used gestures? When and how?
3. Do you often hide or show your feelings? Why?

Part 2
I’m going to give you a topic and I’d like you to talk about it for 2-3 minutes. Before
you talk you have one minute to think about what you’re going to say. You can make
notes if you wish.

Talk about the pollution problem in your hometown or in your country.


You should say:
- How the situation is
- What the causes and effects are
- What actions should be taken to deal with this problem

and explain why you think this pollution problem is a serious problem in
your hometown/country.

Part 3
We’ve been talking about problems. Now I’d like to ask you some more questions about
this topic.
1. How important is it to protect the environment?
2. What do you think of global warming issue?
3. Should all governments in the world work together to protect the environment?

It’s been very interesting talking to you. Thank you and good luck.

59
HANOI UNIVERSITY OF BUSINESS AND TECHNOLOGY
FACULTY OF ENGLISH LANGUAGE STUDIES
*****

3.1 SPEAKING TEST

Topic 5

Student –Examiner (suggested questions)


Good morning/ afternoon.
Please take a seat. My name is … and I’m your examiner. Can you tell me your full
name, please?
What should I call you?

Part 1: Now, I’d like to ask you some questions your opinions and experiences about
crimes.
1. Have you ever witnessed a crime? When and how?
2. Which crime do you think the most and the least serious?
3. What punishment do you think is the most effective to each criminal?

Part 2
I’m going to give you a topic and I’d like you to talk about it for 2-3 minutes. Before
you talk you have one minute to think about what you’re going to say. You can make
notes if you wish.

Talk about the problem of drunkenness in your country. You should


say:
- How the situation is
- What the effects are
- What should be done to change the situation

and explain why you think drunkenness is a serious social problem in your
country.

Part 3
We’ve been talking about drunkenness. Now I’d like to ask you some more questions
about this topic.
1. What do you think are the common reasons that people often use alcohol?
2. Do you agree that drinking is a good way to deal with stress?
3. Is it a good idea to limit the amount of alcohol a person can buy? Why (not)?

It’s been very interesting talking to you. Thank you and good luck.

60
HANOI UNIVERSITY OF BUSINESS AND TECHNOLOGY
FACULTY OF ENGLISH LANGUAGE STUDIES
*****

3.1 SPEAKING TEST

Topic 6

Student –Examiner (suggested questions)


Good morning/ afternoon.
Please take a seat. My name is … and I’m your examiner. Can you tell me your full
name, please?
What should I call you?

Part 1: Now, I’d like to ask you some questions about your favorite movie.
1. What is your favorite kind of movies?
2. Do you like movies about crimes? Why (not)?
3. Why do you think movies about crimes are so popular on TV?

Part 2
I’m going to give you a topic and I’d like you to talk about it for 2-3 minutes. Before
you talk you have one minute to think about what you’re going to say. You can make
notes if you wish.

Talk about the problem of divorce in your country. You should say:
- How the situation is
- What the causes and effects are
- What should be done to reduce the divorce rate

and explain why you think divorce problem is becoming more and more
serious in your country.

Part 3
We’ve been talking about divorce issue. Now I’d like to ask you some more questions
related to this topic.
1. Do you think that getting married is a must to everyone?
2. What do you think of the trend of cohabiting without marriage?
3. Is it a good idea to get married again after a divorce? Why (not)?

It’s been very interesting talking to you. Thank you and good luck.

61
HANOI UNIVERSITY OF BUSINESS AND TECHNOLOGY
FACULTY OF ENGLISH LANGUAGE STUDIES
*****

3.1 SPEAKING TEST


Topic 7

Student –Examiner (suggested questions)


Good morning/ afternoon.
Please take a seat. My name is … and I’m your examiner. Can you tell me your full
name, please?
What should I call you?

Part 1 : Now, I’d like to ask you some questions about your opinions of art.
1. Do you often go to an art gallery? Why (not)?
2. Do you like painting or drawing?
3. Who is your favorite painter?

Part 2
I’m going to give you a topic and I’d like you to talk about it for 2-3 minutes. Before
you talk you have one minute to think about what you’re going to say. You can make
notes if you wish.

Give your opinion on cosmetic surgery issue. You should say:


- If you are For or Against
- How popular it is at present
- What the benefits (if you are For) or harms (if you are Against) of
cosmetic surgery

and explain why more and more people want to get cosmetic surgery.

Part 3
We’ve been talking about cosmetic surgery. Now I’d like to ask you some more
questions about this topic.
1. How important is the appearance beauty to women?
2. People say that “the beauty is in the eye of the beholder”. Do you agree?
3. Which is more important: appearance or personality?

It’s been very interesting talking to you. Thank you and good luck.

62
HANOI UNIVERSITY OF BUSINESS AND TECHNOLOGY
FACULTY OF ENGLISH LANGUAGE STUDIES
*****

3.1 SPEAKING TEST

Topic 8

Student –Examiner (suggested questions)


Good morning/ afternoon.
Please take a seat. My name is … and I’m your examiner. Can you tell me your full
name, please?
What should I call you?

Part 1. Now, I’d like to ask you some questions about your life experiences.
1. Do you think you are a lucky or unlucky person?
2. Do you think that success needs some luck in it?
3. Have you ever had any lucky or unlucky experiences so far?

Part 2
I’m going to give you a topic and I’d like you to talk about it for 2-3 minutes. Before
you talk you have one minute to think about what you’re going to say. You can make
notes if you wish.

Give you opinion on the statement: “Social networking sites are a waste
of time”. You should say:
- If you agree or disagree
- How people are using social networking sites
- What the good points or bad points of social networking sites are

and explain how social networking sites have changed human life.

Part 3
We’ve been talking about social networks. Now I’d like to ask you some more questions
about this topic.
1. How do you use social networking sites?
2. How much time a day do you think is suitable for using social networking sites?
3. Can you list out some uses of social networking sites in education?

It’s been very interesting talking to you. Thank you and good luck.

63
HANOI UNIVERSITY OF BUSINESS AND TECHNOLOGY
FACULTY OF ENGLISH LANGUAGE STUDIES
*****

3.1 SPEAKING TEST

Topic 9

Student –Examiner (suggested questions)


Good morning/ afternoon.
Please take a seat. My name is … and I’m your examiner. Can you tell me your full
name, please?
What should I call you?

Part 1. Now, I’d like to ask you some questions about your school day.
1. How many classes do you have in a day?
2. How are the classes at university different from the ones at high school?
3. Which class do you like to be in the most?

Part 2
I’m going to give you a topic and I’d like you to talk about it for 2-3 minutes. Before
you talk, you have one minute to think about what you’re going to say. You can make
notes if you wish.

Give you opinion on the statement: “What you wear is not important”.
You should say:
- If you agree or disagree
- Why you agree or disagree
- What makes a person more impressive in front of others

and give some examples to support your opinion.

Part 3
We’ve been talking about dressing and appearance. Now I’d like to ask you some more
questions about this topic.
1. How do you think clothes can reflect someone’s personalities?
2. Which is more important: comfort or fashion?
3. Who do you think cares more about clothes: men or women?

It’s been very interesting talking to you. Thank you and good luck.

64
HANOI UNIVERSITY OF BUSINESS AND TECHNOLOGY
FACULTY OF ENGLISH LANGUAGE STUDIES
*****

3.1 SPEAKING TEST

Topic 10

Student –Examiner (suggested questions)


Good morning/ afternoon.
Please take a seat. My name is … and I’m your examiner. Can you tell me your full
name, please?
What should I call you?

Part 1. Now, I’d like to ask you some questions about memory.
1. What kinds of things are hard/easy for you to remember?
2. How do you usually remember things? Do you often use a special device to
remember things?
3. Do you agree that remembering is sometimes more important than intelligence?

Part 2
I’m going to give you a topic and I’d like you to talk about it for 2-3 minutes. Before
you talk, you have one minute to think about what you’re going to say. You can make
notes if you wish.

Describe a good habit you have. You should say:


- What the habit is
- When you started getting it
- What benefits it has brought back to you

And explain how the habit has changed your life.

Part 3
We’ve been talking about habit. Now I’d like to ask you some more questions about this
topic.
1. People say that “habits create personality”. Do you agree?
2. What do people need to do to maintain a good habit?
3. What should adults do to promote good habits among children?

It’s been very interesting talking to you. Thank you and good luck.

65
HANOI UNIVERSITY OF BUSINESS AND TECHNOLOGY
FACULTY OF ENGLISH LANGUAGE STUDIES
*****

3.1 SPEAKING TEST

Topic 11

Student –Examiner (suggested questions)


Good morning/ afternoon.
Please take a seat. My name is … and I’m your examiner. Can you tell me your full
name, please?
What should I call you?

Part 1: Now, I’d like to ask you some questions about problems.
1. Do you have any problems in your life?
2. How do you overcome these problems?
3. What do you do if you fail in the speaking test?

Part 2: I’m going to give you a topic and I’d like you to talk about it for 2-3 minutes.
Before you talk, you have one minute to think about what you’re going to say. You can
make notes if you wish.
Describe a person who solved a problem in a smart way. You should say:
- Who this person is?
- What the problem was?
- How he/she solved it?

and explain why you think he/ she did it in a smart way?

Part 3: We’ve been talking about a person who solved a problem in a smart way. Now
I’d like to ask you some more questions about this topic.

1. Do you think children are born smart or do they learn to become smart?
2. How do children become smart at school?
3. Why does modern society need talent of all kinds?

It’s been very interesting talking to you. Thank you and good luck.

66
HANOI UNIVERSITY OF BUSINESS AND TECHNOLOGY
FACULTY OF ENGLISH LANGUAGE STUDIES
*****

3.1 SPEAKING TEST

Topic 12

Student –Examiner (suggested questions)


Good morning/ afternoon.
Please take a seat. My name is … and I’m your examiner. Can you tell me your full
name, please?
What should I call you?

Part 1: Now, I’d like to ask you some questions about your interests.
1. Do you like reading books or watching movies?
2. What kind of book do you prefer?
3. Do you think ebook will replace paper book in the near future?

Part 2
I’m going to give you a topic and I’d like you to talk about it for 2-3 minutes. Before
you talk, you have one minute to think about what you’re going to say. You can make
notes if you wish.

Describe an exciting book that you enjoy reading. You should say:
- When you read it?
- What kind of book it is?
- What is about?

and explain why you think it is exciting?

Part 3
We’ve been talking about your exciting book. Now I’d like to ask you some more
questions about this topic.

1. Do you think it is important to read the book before watching the movie version
of it?
2. Do boys and girls like the same kinds of books?
3. What kind of books do you think young Vietnamese people like to read?

It’s been very interesting talking to you. Thank you and good luck.

67
HANOI UNIVERSITY OF BUSINESS AND TECHNOLOGY
FACULTY OF ENGLISH LANGUAGE STUDIES
*****

3.1 SPEAKING TEST

Topic 13

Student –Examiner (suggested questions)


Good morning/ afternoon.
Please take a seat. My name is … and I’m your examiner. Can you tell me your full
name, please?
What should I call you?

Part 1: Now, I’d like to ask you some questions about crime.
1. Do you like playing sports? Why? Why not?
2. Can you name some types of sports?
3. Do you like team sport or individual sport? Why? Why not?

Part 2
I’m going to give you a topic and I’d like you to talk about it for 2-3 minutes. Before
you talk, you have one minute to think about what you’re going to say. You can make
notes if you wish.

Describe a famous athlete you know. You should say:

- Who he/she is
- How you know him/her
- What he/she has achieved

and explain why he/she is famous.

Part 3
We’ve been talking about a famous athlete. Now I’d like to ask you some more
questions about this topic.

1. What kinds of exercises do young Vietnamese people like?


2. What characteristics do you think an athlete should have?
3. Why are challenges do you think athletes face?

It’s been very interesting talking to you. Thank you and good luck.

68
HANOI UNIVERSITY OF BUSINESS AND TECHNOLOGY
FACULTY OF ENGLISH LANGUAGE STUDIES
*****
3.1 SPEAKING TEST

Topic 14

Student –Examiner (suggested questions)


Good morning/ afternoon.
Please take a seat. My name is … and I’m your examiner. Can you tell me your full
name, please?
What should I call you?

Part 1: Now, I’d like to ask you some questions about luck.
1. How superstitious are you?
2. Do you have a lucky or unlucky number / colour / day / food?
3. What do you do if you want to be lucky?

Part 2: I’m going to give you a topic and I’d like you to talk about it for 2-3 minutes.
Before you talk, you have one minute to think about what you’re going to say. You can
make notes if you wish.

Describe a time you felt really lucky. You should say:

- When it was
- What happened at that time
- Who or what involved

and explain why you think you were really lucky.

Part 3: We’ve been talking about luck. Now I’d like to ask you some more questions
about this topic.

1. Do you think some people are naturally lucky or unlucky?


2. What does it mean to be lucky?
3. There is a statement, ‘A real man makes his own luck.’ – Do you agree?

It’s been very interesting talking to you. Thank you and good luck.

69
HANOI UNIVERSITY OF BUSINESS AND TECHNOLOGY
FACULTY OF ENGLISH LANGUAGE STUDIES
*****

3.1 SPEAKING TEST

Topic 15

Student –Examiner (suggested questions)


Good morning/ afternoon.
Please take a seat. My name is … and I’m your examiner. Can you tell me your full
name, please?
What should I call you?

Part 1: Now, I’d like to ask you some questions about your English learning.
1. How long have you been learning English?
2. How much time do you spend learning English a day?
3. What language skill do you find the most difficult to learn?

Part 2
I’m going to give you a topic and I’d like you to talk about it for 2-3 minutes. Before
you talk, you have one minute to think about what you’re going to say. You can make
notes if you wish.

Describe your English class. You should say:

- How often your English class is


- What activities you often do in the class
- What your favourite activity is in the class

and explain if you like your English class or not and why.

Part 3
We’ve been talking about your English class. Now I’d like to ask you some more
questions about this topic.

1. What can be the challenges when learning English in a large class with more
than 20 students?
2. What should students do to learn English well in class?
3. Where do you prefer to study English: in class or outside class? Why?

It’s been very interesting talking to you. Thank you and good luck.

70
HANOI UNIVERSITY OF BUSINESS AND TECHNOLOGY
FACULTY OF ENGLISH LANGUAGE STUDIES
*****

3.1 SPEAKING TEST

Topic 16

Student – Examiner (suggested questions)


Good morning/ afternoon.
Please take a seat. My name is … and I’m your examiner. Can you tell me your full
name, please?
What should I call you?

Part 1: Now, I’d like to ask you some questions about crime.
1. How often do you read crime stories?
2. Can you name some types of crime?
3. Is it safe in the place where you are living now?

Part 2
I’m going to give you a topic and I’d like you to talk about it for 2-3 minutes. Before
you talk, you have one minute to think about what you’re going to say. You can make
notes if you wish.

Talk about a crime story that you know. You should say:

- What type of crime it was


- When it happened
- What happened and what the consequences were

and explain how you felt about that crime story.

Part 3
We’ve been talking about crime. Now I’d like to ask you some more questions about this
topic.

1. Why do you think crime stories are popular on TV and in the movies now?
2. What do you think about the statement: ‘Once a criminal, always a criminal.’?
3. In your opinion, what can be the causes of crime?

It’s been very interesting talking to you. Thank you and good luck.

71
HANOI UNIVERSITY OF BUSINESS AND TECHNOLOGY
FACULTY OF ENGLISH LANGUAGE STUDIES
*****

3.1 SPEAKING TEST

Topic 17: Jobs

Student – Examiner (suggested questions)


Good morning/ afternoon.
Please take a seat. My name is … and I’m your examiner. Can you tell me your full
name, please?
What should I call you?

Part 1: Now, I’d like to ask you some questions about jobs.
1. How long have you lived in Hanoi?
2. Do you have a part-time job?
3. Should university students focus on studying only or should they have a part time
job?

Part 2: I’m going to give you a topic and I’d like you to talk about it for 2-3 minutes.
Before you talk, you have one minute to think about what you’re going to say. You can
make notes if you wish.

Describe a job you would like to have in the future. You should say:

- What the job is


- Where you will work at
- What you will need to do this job

and explain why you want to do this job.

Part 3: We’ve been talking about jobs. Now I’d like to ask you some more questions
about this topic.
1. What is more important to you for the future job, salary or job satisfaction?
2. Would you like to come back to your hometown to work or would you stay in
Hanoi? Why?
3. Why do you think many people have moved their hometown to big cities to find
a job?

It’s been very interesting talking to you. Thank you and good luck.

72
HANOI UNIVERSITY OF BUSINESS AND TECHNOLOGY
FACULTY OF ENGLISH LANGUAGE STUDIES
*****

3.1 SPEAKING TEST

Topic 18

Student – Examiner (suggested questions)


Good morning/ afternoon.
Please take a seat. My name is … and I’m your examiner. Can you tell me your full
name, please?
What should I call you?

Part 1: Now, I’d like to ask you some questions about TV programs.
1. How much time per day do you spend watching TV?
2. Are TV programs useful for you?
3. What are popular TV programs recently?

Part 2: I’m going to give you a topic and I’d like you to talk about it for 2-3 minutes.
Before you talk, you have one minute to think about what you’re going to say. You can
make notes if you wish.

Describe your favorite TV program. You should say:

- What kind of TV program it is


- How often you watch it
- What the TV program is about

and explain why you like it.

Part 3: We’ve been talking about TV programs. Now I’d like to ask you some more
questions about this topic.
1. Do you prefer watching films at home or in the cinema?
1. What do you think about Vietnamese drama/films shown on TV recently?
2. How have films changed over years?

It’s been very interesting talking to you. Thank you and good luck.

73
HANOI UNIVERSITY OF BUSINESS AND TECHNOLOGY
FACULTY OF ENGLISH LANGUAGE STUDIES
*****

3.1 SPEAKING TEST

Topic 19: TRAVEL

Student – Examiner (suggested questions)


Good morning/ afternoon.
Please take a seat. My name is … and I’m your examiner. Can you tell me your full
name, please?
What should I call you?

Part 1: Now, I’d like to ask you some questions about travel.
1. Do you like travelling?
2. What is the best place you have been to in Vietnam?
3. Have you ever been abroad? / Have you travelled to a foreign country?

Part 2: I’m going to give you a topic and I’d like you to talk about it for 2-3 minutes.
Before you talk, you have one minute to think about what you’re going to say. You can
make notes if you wish.

Describe a foreign country where you want to visit in the future. You
should say:

- Which country you want to travel to


- When you will travel
- What you will do while you are travelling there

and explain why you want to travel to this place.

Part 3: We’ve been talking about travel. Now I’d like to ask you some more questions
about this topic.
1. What are the most popular travelling destinations in your country?
2. How has people’s way of travelling changed?
3. How can travelling help you?

It’s been very interesting talking to you. Thank you and good luck.

74
HANOI UNIVERSITY OF BUSINESS AND TECHNOLOGY
FACULTY OF ENGLISH LANGUAGE STUDIES
*****

3.1 SPEAKING TEST

Topic 20

Student – Examiner (suggested questions)


Good morning/ afternoon.
Please take a seat. My name is … and I’m your examiner. Can you tell me your full
name, please?
What should I call you?

Part 1: Now, I’d like to ask you some questions about neighbourhood.
1. Which family member do you spend the most time with?
2. Do you like your neighbourhood?
3. Is there anything you would like to change about your neighbourhood?

Part 2: I’m going to give you a topic and I’d like you to talk about it for 2-3 minutes.
Before you talk, you have one minute to think about what you’re going to say. You can
make notes if you wish.

Describe your favourite television show. You should say:

- What it is
- What channel it is on
- When it is on
and explain why you like it so much.

Part 3: We’ve been talking about your favourite television show. Now I’d like to ask you
some more questions about this topic.
1. Do you spend a lot of time watching television?
2. What do you think about advertising on television; does it influence you?
3. Why do you think watching television is such a popular pastime?

It’s been very interesting talking to you. Thank you and good luck.

75
HANOI UNIVERSITY OF BUSINESS AND TECHNOLOGY
FACULTY OF ENGLISH LANGUAGE STUDIES
*****

3.1 SPEAKING TEST

Topic 21

Student – Examiner (suggested questions)


Good morning/ afternoon.
Please take a seat. My name is … and I’m your examiner. Can you tell me your full
name, please?
What should I call you?

Part 1: Now, I’d like to ask you some questions about your studying.
1. What is the most important thing that you’ve learned at school?
2. Is there something new you’d really like to learn about? Why?
3. Do you plan to study further in the future? (What do you want to study?)

Part 2: I’m going to give you a topic and I’d like you to talk about it for 2-3 minutes.
Before you talk, you have one minute to think about what you’re going to say. You can
make notes if you wish.

Describe the recent excursion that impressed you most. You should say:

- What it was
- When it took place
- Who you went with

and explain why it impressed you most.

Part 3: We’ve been talking about your recent excursion. Now I’d like to ask you some
more questions about this topic.
1. Do people of different ages always have different interests?
2. What kinds of activities do you enjoy doing with your parents?
3. When you go out with your friends, how do you decide what to do?

It’s been very interesting talking to you. Thank you and good luck.

76
HANOI UNIVERSITY OF BUSINESS AND TECHNOLOGY
FACULTY OF ENGLISH LANGUAGE STUDIES
*****

3.1 SPEAKING TEST

Topic 22

Student – Examiner (suggested questions)


Good morning/ afternoon.
Please take a seat. My name is … and I’m your examiner. Can you tell me your full
name, please?
What should I call you?

Part 1: Now, I’d like to ask you some questions about your travelling.
1. What is your favourite way of travelling? Why?
2. How do you pass the time on a long journey?
3. Where do you think you’ll spend your holidays next year?

Part 2: I’m going to give you a topic and I’d like you to talk about it for 2-3 minutes.
Before you talk, you have one minute to think about what you’re going to say. You can
make notes if you wish.

Describe a sporting activity that you like doing the most. You should say:

- What it is
- Where and when you can do it
- Who you like doing it with

and explain why you like it so much.

Part 3: We’ve been talking about your favourite sporting activity. Now I’d like to ask
you some more questions about this topic.
1. Do you think the sport has an age limit, or should everyone be able to take part
in?
2. In the sports world, is winning all that matters?
3. Why do you think sportspeople feel the need to take performance-enhancing
drugs?

It’s been very interesting talking to you. Thank you and good luck.

77
HANOI UNIVERSITY OF BUSINESS AND TECHNOLOGY
FACULTY OF ENGLISH LANGUAGE STUDIES
*****

3.1 SPEAKING TEST

Topic 23

Student – Examiner (suggested questions)


Good morning/ afternoon.
Please take a seat. My name is … and I’m your examiner. Can you tell me your full
name, please?
What should I call you?

Part 1: Now, I’d like to ask you some questions about sports.
1. Are you interested in sports?
2. Which do you prefer, football, badminton or table tennis?
3. Do you work out often?

Part 2: I’m going to give you a topic and I’d like you to talk about it for 2-3 minutes.
Before you talk, you have one minute to think about what you’re going to say. You can
make notes if you wish.

Describe your favourite sports. You should say:

- what sport it is
- how often you play it or watch it
- how you play it

and explain why this is your favourite sport.

Part 3: We’ve been talking about your favourite sport. Now I’d like to ask you some
more questions about this topic.
1. Do you think it’s important for students at university to do sports? Why? / Why
not?
2. What could governments do to encourage young people to do more sport and
exercise?
3. When you play a sport, does it matter if you win or lose? Why? / Why not?

It’s been very interesting talking to you. Thank you and good luck.

78
HANOI UNIVERSITY OF BUSINESS AND TECHNOLOGY
FACULTY OF ENGLISH LANGUAGE STUDIES
*****

3.1 SPEAKING TEST

Topic 24

Student – Examiner (suggested questions)


Good morning/ afternoon.
Please take a seat. My name is … and I’m your examiner. Can you tell me your full
name, please?
What should I call you?

Part 1: Now, I’d like to ask you some questions about your work.
1. Would you prefer to work indoors or outdoors? (Why?)
2. Do you prefer working on your own or with other people? (Why?)
3. What kind of work would you like to do in the future? (Why?)

Part 2: I’m going to give you a topic and I’d like you to talk about it for 2-3 minutes.
Before you talk, you have one minute to think about what you’re going to say. You can
make notes if you wish.

Describe the dream job you wish to have. You should say:

- what the job is


- what it involves
- why you have chosen it

and explain why you are interested in this job.

Part 3: We’ve been talking about your dream job. Now I’d like to ask you some more
questions about this topic.
1. Which jobs would you say are most respected in your country?
2. Is money always the most important thing when choosing a job?
3. What do you think would be the most difficult job to do?

It’s been very interesting talking to you. Thank you and good luck.

79
HANOI UNIVERSITY OF BUSINESS AND TECHNOLOGY
FACULTY OF ENGLISH LANGUAGE STUDIES
*****

3.1 SPEAKING TEST

Topic 25

Student – Examiner (suggested questions)


Good morning/ afternoon.
Please take a seat. My name is … and I’m your examiner. Can you tell me your full
name, please?
What should I call you?

Part 1: Now, I’d like to ask you some questions about your living place.
1. What kind of housing/accommodation do you live in?
2. Who do you live with?
3. How long have you lived there?

Part 2: I’m going to give you a topic and I’d like you to talk about it for 2-3 minutes.
Before you talk, you have one minute to think about what you’re going to say. You can
make notes if you wish.

Describe your hometown. You should say:

- What your hometown is like


- What is special about it
- What the people in your hometown are like

and compare it to other cities in the world

Part 3: We’ve been talking about your hometown. Now I’d like to ask you some more
questions about this topic.
1. Do you think it is more difficult living in the countryside or in a big city? Why?
2. Would it be easier for a foreign visitor to fit in a city or a village?
3. Where do you think most people will live in the future?

It’s been very interesting talking to you. Thank you and good luck.

80
HANOI UNIVERSITY OF BUSINESS AND TECHNOLOGY
FACULTY OF ENGLISH LANGUAGE STUDIES
*****
3.1 SPEAKING TEST

Topic 26

Student – Examiner (suggested questions)


Good morning/ afternoon.
Please take a seat. My name is … and I’m your examiner. Can you tell me your full
name, please?
What should I call you?

Part 1: Now, I’d like to ask you some questions about your personal information.
1. What do you do? Do you work or are you a student?
2. Would you like to learn anything new in the future?
3. Is there a job you would really like to do in the future?

Part 2: I’m going to give you a topic and I’d like you to talk about it for 2-3 minutes.
Before you talk, you have one minute to think about what you’re going to say. You can
make notes if you wish.

Describe a job you would like to have. You should say:

- what the job is


- what you know about that job
- what study or training you would need to do so you can get that job

and explain why you would like to have that job.

Part 3: We’ve been talking about your hometown. Now I’d like to ask you some more
questions about this topic.
1. Would you enjoy doing a job of a doctor/ nurse/ chef/ etc.? Why (not)?
2. Which parts of that job would you enjoy more and which would you enjoy less?
3. Do you think being a waiter is a good holiday job for a teenager? Why? / Why
not?

It’s been very interesting talking to you. Thank you and good luck.

81
HANOI UNIVERSITY OF BUSINESS AND TECHNOLOGY
FACULTY OF ENGLISH LANGUAGE STUDIES
*****

3.1 SPEAKING TEST

Topic 27

Student – Examiner (suggested questions)


Good morning/ afternoon.
Please take a seat. My name is … and I’m your examiner. Can you tell me your full
name, please?
What should I call you?

Part 1: Now, I’d like to ask you some questions about your free time.
1. What do you like doing when you have free time?
2. How important is it for you to have free time?
3. Do you think you'll have more free time in the future than you do now?

Part 2: I’m going to give you a topic and I’d like you to talk about it for 2-3 minutes.
Before you talk, you have one minute to think about what you’re going to say. You can
make notes if you wish.

Talk about a new sport or hobby you would like to take up. You should
say:
- what it is
- how you would start it
- why you have not started it yet

and explain what help you would need to take it up.

Part 3: We’ve been talking about your new sport or hobby. Now I’d like to ask you some
more questions about this topic.
1. Do you generally enjoy trying new things? Why?
2. Is there any other activity you would like to try one day? What is it?
3. Who do you think usually likes trying new things - young people or older people?
Why?

It’s been very interesting talking to you. Thank you and good luck.

82
HANOI UNIVERSITY OF BUSINESS AND TECHNOLOGY
FACULTY OF ENGLISH LANGUAGE STUDIES
*****

3.1 SPEAKING TEST

Topic 28

Student – Examiner (suggested questions)


Good morning/ afternoon.
Please take a seat. My name is … and I’m your examiner. Can you tell me your full
name, please?
What should I call you?

Part 1: Now, I’d like to ask you some questions about your preference of living places.
1. Do you prefer to live in the countryside or in the city?
2. What are the disadvantages of living in the countryside?
3. Do you think people damage the environment more when they live in a town or
when they live-in the countryside?

Part 2: I’m going to give you a topic and I’d like you to talk about it for 2-3 minutes.
Before you talk, you have one minute to think about what you’re going to say. You can
make notes if you wish.

Describe a place you visited where the air was very clean and fresh. You
should say:

- where this place was


- when and why you visited this place
- who you were with

and explain how you felt visiting a place where the air was so clean and fresh.

Part 3: We’ve been talking about the place you have visited. Now I’d like to ask you
some more questions about this topic.
1. Do you think humans can live in a completely unnatural or artificial
environment?
2. What are some ways people change or affect the environment when they live in
the countryside?
3. Is it possible for people to have high standard of living without seriously
damaging the environment?

It’s been very interesting talking to you. Thank you and good luck.

83
HANOI UNIVERSITY OF BUSINESS AND TECHNOLOGY
FACULTY OF ENGLISH LANGUAGE STUDIES
*****

3.1 SPEAKING TEST

Topic 29

Student – Examiner (suggested questions)


Good morning/ afternoon.
Please take a seat. My name is … and I’m your examiner. Can you tell me your full
name, please?
What should I call you?

Part 1: Now, I’d like to ask you some questions about music.
1. What types of music do you like listening to?
2. Can you play any musical instruments?
3. Do you think it’s important for children to learn to play a musical instrument?

Part 2: I’m going to give you a topic and I’d like you to talk about it for 2-3 minutes.
Before you talk, you have one minute to think about what you’re going to say. You can
make notes if you wish.

Describe a song that you like. You should say:

- What its name is


- What kind of music it is
- Where/ When you first heard it

and explain why you like it so much.

Part 3: We’ve been talking about the song you like. Now I’d like to ask you some more
questions about this topic.
1. What do you think old people like to listen to?
2. What kinds of music do (small) children in your country like to listen to?
3. Why do you think people of different age groups have different favourite songs?

It’s been very interesting talking to you. Thank you and good luck.

84
HANOI UNIVERSITY OF BUSINESS AND TECHNOLOGY
FACULTY OF ENGLISH LANGUAGE STUDIES
*****

3.1 SPEAKING TEST

Topic 30

Student – Examiner (suggested questions)


Good morning/ afternoon.
Please take a seat. My name is … and I’m your examiner. Can you tell me your full
name, please?
What should I call you?

Part 1: Now, I’d like to ask you some questions about emails.
1. How often do you use email?
2. What do most people in your country use email for?
3. What are the advantages and disadvantages of email?

Part 2: I’m going to give you a topic and I’d like you to talk about it for 2-3 minutes.
Before you talk, you have one minute to think about what you’re going to say. You can
make notes if you wish.

Describe an important email you have received. You should say:

- Who sent it to you


- When you got it
- What it was about

and explain why this email is important to you.

Part 3: We’ve been talking about the important email to you. Now I’d like to ask you
some more questions about this topic.
1. What kinds of people use emails?
2. What effects, if any, do you think the use of emails might have on traditional
culture?
3. Do you think writing emails has strengthened or weakened people’s writing
skills?

It’s been very interesting talking to you. Thank you and good luck.

85
HANOI UNIVERSITY OF BUSINESS AND TECHNOLOGY
FACULTY OF ENGLISH LANGUAGE STUDIES
*****

3.1 SPEAKING TEST

Topic 31

Student – Examiner (suggested questions)


Good morning/ afternoon.
Please take a seat. My name is … and I’m your examiner. Can you tell me your full
name, please?
What should I call you?

Part 1: Now, I’d like to ask you some questions about games.
1. Do you enjoy playing games?
2. What kind of games do children play in your country?
3. Do you think adults should play games?

Part 2: I’m going to give you a topic and I’d like you to talk about it for 2-3 minutes.
Before you talk, you have one minute to think about what you’re going to say. You can
make notes if you wish.

Describe a game children play in your country. You should say:

- What it is
- Where it can be played
- What you can learn from the game

and explain how it is played.

Part 3: We’ve been talking about the game children play in your country. Now I’d like
to ask you some more questions about this topic.
1. What do you think of benefits of games for children?
2. Nowadays children have less and less time for games, what do you think?
3. How has the types of games children play changed in recent years?

It’s been very interesting talking to you. Thank you and good luck.

86
ANSWER KEY
PART 1 – VOCABULARY

1A 2B 3C 4D 5B 6A 7A 8D 9C 10C

11B 12A 13C 14A 15D 16A 17C 18A 19C 20B

21C 22A 23B 24C 25B 26D 27A 28B 29C 30D

31A 32C 33B 34D 35A 36C 37B 38A 39B 40C

41B 42B 43D 44B 45A 46B 47A 48D 49B 50C

51C 52B 53C 54D 55A 56D 57C 58C 59D 60A

61C 62B 63A 64C 65B 66A 67C 68D 69A 70C

71A 72D 73D 74A 75C 76C 77B 78D 79A 80A

81D 82D 83D 84D 85D 86D 87D 88D 89D 90D

91C 92C 93C 94C 95C 96C 97C 98C 99C 100C

101B 102B 103B 104B 105B 106B 107B 108A 109A 110A

110A 112A 113A 114A 115A 116A 117A 118A 119B 120A

121C 122A 123A 124B 125A 126C 127A 128B 129B 130C

131C 132C 133A 134B 135B 136B 137A 138C 139A 140B

141B 142A 143A 144C 145C 146A 147C 148B 149A 150B

151C 152A 153A 154C 155A 156B 157A 158B 159C 160B

161A 162B 163C 164A 165B 166C 167C 168B 169C 170B

87
PART 2 – GRAMMAR

1A 2B 3D 4A 5B 6B 7C 8A 9C 10B

11C 12B 13A 14A 15C 16A 17B 18A 19C 20A

21A 22C 23B 24B 25D 26A 27D 28B 29A 30C

31D 32A 33C 34B 35A 36C 37D 38D 39B 40C

41C 42D 43D 44A 45C 46B 47A 48D 49A 50D

51C 52B 53B 54A 55D 56C 57A 58B 59D 60A

61C 62D 63B 64C 65C 66B 67A 68B 69A 70A

71C 72C 73A 74C 75D 76B 77B 78C 79A 80C

81B 82C 83B 84A 85B 86C 87C 88D 89B 90B

91C 92C 93A 94C 95A 96C 97D 98D 99C 100B

101C 102A 103B 104B 105A 106D 107C 108B 109A 110A

111A 112B 113A 114A 115C 116B 117C 118B 119B 120C

121C 122D 123B 124A 125C 126D 127A 128C 129A 130C

131B 132D 133B 134C 135B 136C 137A 138C 139B 140B

141C 142B 143B 144A 145B 146A 147A 148A 149D 150C

88
PART 3 – READING
Part I

Passage 1
1B 2C 3A 4D 5A 6B 7C 8D 9A 10C

Passage 2
1D 2B 3C 4D 5A 6C 7B 8A 9D 10A

Passage 3
1A 2B 3D 4C 5A 6C 7D 8C 9A 10B

Passage 4
1B 2A 3C 4A 5B 6C 7D 8A 9C 10D

Passage 5
1C 2B 3A 4A 5D 6B 7A 8C 9D 10D

Passage 6
1C 2B 3A 4A 5D 6B 7A 8C 9D 10D

Passage 7
1B 2A 3D 4B 5C 6C 7B 8C 9A 10B

Passage 8
1D 2B 3D 4B 5B 6A 7A 8B 9A 10D

Passage 9
1D 2A 3C 4B 5A 6D 7B 8C 9A 10B

Passage 10
1D 2A 3B 4A 5C 6B 7D 8B 9A 10B

89
PART 3 – READING
Part II

Passage 1 1B 2C 3D 4A 5B
Passage 2 1A 2D 3B 4D 5A
Passage 3 1D 2B 3C 4B 5A
Passage 4 1B 2D 3A 4A 5D
Passage 5 1C 2A 3B 4D 5A
Passage 6 1B 2C 3D 4B 5A
Passage 7 1B 2A 3B 4D 5A
Passage 8 1B 2C 3B 4C 5D
Passage 9 1D 2B 3C 4A 5D
Passage 10 1B 2C 3D 4B 5D
Passage 11 1C 2B 3C 4C 5D
Passage 12 1C 2C 3C 4B 5B
Passage 13 1B 2A 3D 4C 5C
Passage 14 1D 2A 3B 4C 5C
Passage 15 1A 2C 3B 4B 5A
Passage 16 1B 2A 3C 4D 5B
Passage 17 1A 2C 3D 4C 5A
Passage 18 1D 2B 3C 4B 5D
Passage 19 1D 2C 3A 4A 5B
Passage 20 1A 2C 3B 4D 5A
Passage 21 1A 2B 3A 4D 5B
Passage 22 1B 2D 3C 4B 5B
Passage 23 1B 2D 3B 4C 5A
Passage 24 1B 2B 3C 4A 5B
Passage 25 1B 2A 3C 4B 5A
Passage 26 1B 2C 3A 4B 5B
Passage 27 1B 2C 3C 4B 5D
Passage 28 1B 2A 3A 4B 5A

90
PART 4 – LISTENING
Part I

Recording 1 1B 2C 3C 4A 5D
Recording 2 1B 2A 3C 4D 5B
Recording 3 1A 2C 3D 4B 5C
Recording 4 1B 2D 3A 4B 5C
Recording 5 1B 2D 3C 4A 5B
Recording 6 1B 2A 3D 4A 5C
Recording 7 1B 2A 3D 4B 5C
Recording 8 1A 2C 3A 4D 5C
Recording 9 1C 2A 3B 4C 5D
Recording 10 1B 2D 3A 4C 5A
Recording 11 1B 2D 3B 4D 5A
Recording 12 1B 2A 3D 4C 5B
Recording 13 1A 2C 3C 4B 5D
Recording 14 1D 2A 3C 4D 5C
Recording 15 1D 2A 3A 4B 5C
Recording 16 1D 2C 3A 4B 5D
Recording 17 1B 2C 3A 4A 5D
Recording 18 1D 2D 3B 4A 5D
Recording 19 1B 2D 3C 4A 5C

91
PART 4 – LISTENING
Part II

Recording 1 Recording 2 Recording 3 Recording 4 Recording 5


1. attention 1. thousand 1. humour 1. draw 1. miserable
2. important 2. animals 2. relationships 2. sequence 2. smile
3. remember 3. months 3. rapport 3. Whenever 3. newspaper
4. several 4. solution 4. favourite 4. blocks 4. reaction
5. confident 5. shops 5. embarrassing 5. finished 5. flirting

Recording 6 Recording 7 Recording 8 Recording 9 Recording 10


1. frustrated 1. prepare 1. couple 1. intelligent 1. experience
2. answer 2. vegetables 2. whatever 2. touristy 2. practical
3. online 3. progammed 3. surprise 3. languages 3. academic
4. strange 4. choose 4. airport 4. incredible 4. technical
5. interacting 5. ingredients 5. outside 5. Japanese 5. dives

Recording 11 Recording 12 Recording 13 Recording 14 Recording 15


1. clean and 1. 6,500 1. exactly alike 1. many reasons 1. TGV 740
refreshing kilometers 2. straight black 2. level of 2. free beverages
2. rescue long hair education 3. changing
workers 2. supply 3. look different 3. subjects trains
3. from 3. rainy season 4. different 4. discipline 4. permitted
spreading 4. many animals styles 5. sense of 5. 18 and 25
4. restore the 5. efficient 5. thoughts community
beach
5. safety

Recording 16 Recording 17 Recording 18 Recording 19 Recording 20


1. written tests 1. semester 1. major 1. confused 1. use your
2. five questions 2. uses ideas requirements 2. viewpoint of hands
3. summary 3. chemistry, 2. discuss the time 2. goalkeeper
handout biology content 3. collection 3. fingers
4. final grade 4. geography 3. challenge 4. local climate 4. kicks the ball
5. discuss the class 4. oral 5. changes 5. team player
topics 5. Energy presentation
5. specific

92

You might also like